Site Loader

Содержание

Реферат дисциплина «теоретическая механика» — Автореферат диссертации

ФЕДЕРАЛЬНОЕ АГЕНТСТВО ПО ОБРАЗОВАНИЮ

ФИЛИАЛ ГОУ ВПО МОСКОВСКОГО ГОСУДАРСТВЕННОГО ИНДУСТРИАЛЬНОГО УНИВЕРСИТЕТА В г. ВЯЗЬМЕ СМОЛЕНСКОЙ ОБЛАСТИ

РЕФЕРАТ

Дисциплина: «Теоретическая механика»

Тема: «Понятие момента силы. Теорема Вариньона»

Специальность: 190201 Автомобиле- и тракторостроение

Группа: 06АД11

Студент: Гарский Роман Олегович

Преподаватель: к. т. н. Осипян Валентин Георгиевич.

2007г.

Моменты силы относительно точки и оси.

Для рассмотрения различных систем сил необходимо ввести понятия алгебраического и векторного моментов силы относительно точки и момента силы относительно оси. Введём эти характеристики действия силы на твёрдое тело и рассмотрим их свойства.

Алгебраический момент силы относительно точки.

При рассмотрении плоской системы сил, приложенных к твёрдому телу, используется понятие алгебраического момента силы относительно точки.


Алгебраическим моментом силы относительно точки называют произведение модуля силы на плечо силы относительно этой точки взятое со знаком плюс или минус.

Плечом h силы относительно точки называют кратчайшее расстояние между этой точкой и линией действия силы, т.е. длину отрезка перпендикуляра, опущенного из точки О на линию действия силы . Обозначим Мо() или Мо алгебраический момент силы относительно точки О. Тогда:

Мо() = ±Fh.

Если сила стремится вращать тело вокруг моментной точки (точки, относительно которой вычисляют алгебраический момент силы) против часовой стрелки, то берём знак плюс, если по часовой стрелке – знак минус.

Алгебраический момент силы представляет собой произведение силы на длину. (в СИ Н*м).

Из определения алгебраического момента силы относительно точки следует, что он не зависит от переноса силы вдоль её линии действия. Алгебраический момент силы относительно точки равен нулю, если линия действия силы проходит через моментную точку. Сумма алгебраических моментов относительно точки двух равных по величине, но противоположных по направлению сил, действующих вдоль одной прямой, равна нулю. Численно алгебраический момент относительно точки равен удвоенной площади треугольника, построенного на силе и моментной точке:

Мо() = ±2 пл. ▲± ОАВ.

Векторный момент силы относительно точки.

При рассмотрении пространственной системы сил, приложенных к твёрдому телу, применяется понятие векторного момента силы относительно точки.

Векторным моментом силы относительно точки называют вектор, приложенный в этой точке и равный по модулю произведению силы на плечо силы относительно этой точки. Векторный момент силы направлен перпендикулярно плоскости, в которой лежат сила и моментная точка, таким образом, что с его конца можно видеть стремление силы вращать тело против движения часовой стрелки.

Плечом h силы относительно точки О называют кратчайшее расстояние от этой точки до линии действия силы.

Условимся векторный момент силы относительно точки О обозначать о(), а его числовую величину о(). Тогда, согласно определению,

о()= Fh.

Как и для алгебраического момента, векторный момент силы относительно точки равен удвоенной площади треугольника, построенного на силе и моментной точке:

о() = 2 пл. ▲ ОАВ.

Справедлива формула

о() = × ,

где — радиус-вектор, проведённый из моментной точки О в точку приложения силы или любую другую точку линии действия силы.

Чтобы убедиться в справедливости этой формулы, достаточно показать, что × по величине и по направлению выражает векторный момент силы относительно точки О, если для построения векторного произведения силу перенести параллельно самой себе в точку О. По определению векторного произведения двух векторов известно, что

× = Fr sin (^ ). ) = h, причём это равенство справедливо для любой точки линии действия, куда проведён радиус-вектор . Итак,

× = Fh,

что совпадает с векторным моментом силы относительно точки О. Вектор

× , как известно, перпендикулярен плоскости, в которой расположены векторы и т. е. плоскости треугольника ОАВ, которой перпендикулярен и векторный момент о().

Направление × тоже совпадает с направлением о(). Заметим, что векторный момент силы относительно точки считается вектором, приложенным к этой точке.

Векторный момент силы относительно точки не изменяется от переноса силы вдоль её линии действия. Он станет равным нулю, если линия действия силы пройдёт через моментную точку.

Если сила дана своими проекции Fx, Fy, Fz на оси координат и даны координаты x, y, z точки приложения этой силы, то векторный момент относительно начала координат, согласно формуле после разложения по осям координат вычисляем по формуле:


где , — единичные векторы, направления по осям координат.

Используя формулу, можно выделить проекции о() на оси координат:

Модуль векторного момента о() и косинусы углов его с осями координат определяем по формулам:


В данной системе формул числовую величину вектора о() берём со знаком плюс.

Момент силы относительно оси.

Моментом силы относительно оси называют алгебраический момент проекции этой силы на плоскость, перпендикулярную оси, относительно точки пересечения плоскостью.


Момент силы относительно оси считается положительным, если проекция силы на плоскость, перпендикулярно оси (проекция силы на плоскость является вектором), стремится вращать тело вокруг положительного направления оси против часовой стрелки, и отрицательным, если она стремиться вращать тело по часовой стрелке. Момент силы, например относительно оси Оz обозначим z (). По определению,

где п – вектор проекции силы на плоскость П, перпендикулярную оси Оz, а точка О- точка пересечения оси Оz с плоскостью П.


Из определения момента силы относительно оси следует, что введённый выше алгебраический момент силы относительно точки можно считать моментом силы относительно оси, проходящей через эту точку, перпендикулярно плоскости, в которой лежат сила и моментная точка.

Момент силы относительно оси можно выразить через площадь треугольника, построенного на проекции силы п и точке пересечения О оси с плоскостью:

Из этой формулы можно получить следующие важные свойства момента силы относительно оси.

  1. Момент силы относительно оси равен нулю, если сила параллельна оси. В этом случае равна нулю проекция силы на плоскость, перпендикулярную оси.

  2. Момент силы относительно оси равен нулю, если линия действия силы пересекает эту ось. В этом случае линия действия силы на плоскость, перпендикулярную оси, проходит через точку пересечения оси с плоскостью и, следовательно, равно нулю плечо силы п относительно точки О.

В обоих этих случаях ось и сила лежат в одной плоскости. Объединяя их, можно сказать, что момент силы относительно оси равен нулю, если сила и ось лежат в одной плоскости.

Связь момента силы относительно оси с векторным моментом силы относительно точки на оси.

Формула отражает искомую связь между моментом силы относительно оси и векторными моментами силы относительно точек, лежащих на этой оси: момент силы относительно оси равен проекции на эту ось векторного момента силы относительно любой точки на оси.

Эту зависимость между моментом силы относительно оси и векторным моментом силы относительно точки на оси можно принять за определение момента силы относительно оси.

Формулы для моментов силы относительно осей координат.


Используя связь момента силы относительно оси с векторным мо­ментом силы относительно точки на оси, можно получить формулы для вычисления моментов относительно осей координат, если даны проек­ции силы на оси координат и координаты точки приложения силы.

По этим формулам получаются необходимые знаки для Мх (F), Му (F)y Mz (F), если проекции силы Fx, F у Fz на оси координат и ко­ординаты х, у, z точки приложения силы подставлять в них со знаками этих величин.

При решении задач момент силы относительно оси часто получают, используя его определение, т.е. проецируя силу на плоскость, перпендикулярную оси, и вычисляя затем алгебраический момент этой проекции относительно точки пересечения с этой плоскостью.

Теорема о моменте равнодействующей силы (Теорема Вариньона).

Для случая, когда любая система сил, приложенных к твердому те­лу, плоская или пространственная, приводится к равнодействующей силе, часто применяют так называемую теорему Вариньона: вектор­ный момент равнодействующей рассматриваемой системы сил относи­тельно любой точки равен сумме векторных моментов всех сил этой системы относительно той же точки.


Пусть на твердое тело действует любая система сил (,) имеющая равнодействующую *, т. е.

Добавим к заданной системе сил ее уравновешивающую силу *’, которая равна по модулю, но противоположна по направлению равно­действующей силе * и имеет с ней общую линию действия. Тогда


т.е. при добавлении к системе сил уравновешивающей силы, соглас­но определению уравновешивающей силы, образуется новая система сил, эквивалентная нулю и, следовательно, удовлетворяющая условиям равновесия системы сил, приложенных к твердому телу. В частно­сти, сумма векторных моментов сил этой новой системы сил относительно любой точки 0 равна нулю:

но


так как *’ и *— две равные и противоположно направленные силы, действующие вдоль одной прямой. Подставляя, получаем

откуда следует теорема Вариньона


Момент равнодействующей силы относительно произвольной оси равен сумме моментов сил системы относительно той же оси.

Например для оси Оz: .

Для случая плоской системы сил, если точку О выбрать в плоскости действия сил, получаем:

Это теорема Вариньона для плоской системы сил: алгебраический момент равнодействующей плоской системы сил относительно любой точки, лежащей в плоскости действия сил, равен сумме алгебраических моментов всех сил этой системы относительно той же точки.

Моменты силы относительно точки и оси в теоретической механике

Содержание:

Моменты силы относительно точки и оси:

Для рассмотрения различных систем сил необходимо ввести понятия алгебраического и векторного моментов силы относительно точки и момента силы относительно оси. Введем эти характеристики действия силы на твердое тело и рассмотрим их свойства.

Алгебраический момент силы относительно точки

При рассмотрении плоской системы сил, приложенных к твердому телу, используется понятие алгебраического момента силы относительно точки.

Рис. 19

Алгебраическим моментом силы относительно точки называют произведение модуля силы на плечо силы относительно этой точки (рис. 19), взятое со знаком плюс или минус.

Плечом относительно точки называют кратчайшее расстояние между этой точкой и линией действия силы, т. е. длину отрезка перпендикуляра, опущенного из точки на линию действия силы .

Обозначим или алгебраический момент силы относительно точки . Тогда

Если сила стремится вращать тело вокруг моментной точки (точки, относительно которой вычисляют алгебраический момент силы) против часовой стрелки, то берем знак плюс, если по часовой стрелке — знак минус.

Алгебраический момент силы представляет собой произведение силы на длину (в ).

Из определения алгебраического момента силы относительно точки следует, что он не зависит от переноса силы вдоль ее линии действия. Алгебраический момент силы относительно точки равен нулю, если линия действия силы проходит через моментную точку. Сумма алгебраических моментов относительно точки двух равных по модулю, но противоположных по направлению сил, действующих вдоль одной прямой, равна нулю. Численно алгебраический момент относительно точки равен удвоенной площади треугольника, построенного на силе  и моментной точке:

Векторный момент силы относительно точки

При рассмотрении пространственной системы сил, приложенных к твердому телу, применяется понятие векторного момента силы относительно точки.

Векторным моментом силы относительно точки называют вектор, приложенный в этой точке и равный по модулю произведению силы на плечо силы относительно этой точки. Векторный момент силы направлен перпендикулярно плоскости, в которой лежат сила и моментная точка, таким образом, что с его конца можно видеть стремление силы вращать тело против движения часовой стрелки (рис. 20).

Плечом силы относительно точки называют кратчайшее расстояние от этой точки до линии действия силы.

Рис. 20

Условимся векторный момент  силы относительно точки обозначать , а его числовую величину — . Тогда, согласно определению,

Как и для алгебраического момента, векторный момент силы относительно точки равен удвоенной площади треугольника, построенного на силе и моментной точке:

Справедлива формула

где —радиус-вектор, проведенный из моментной точки в точку приложения силы или любую другую точку линии действия силы.

Чтобы убедиться в справедливости формулы (3), достаточно показать, что по величине и направлению выражает векторный момент силы относительно точки . По определению векторного произведения двух векторов известно, что

Как показано на рис. 20, , причем это равенство справедливо для любой точки линии действия, куда проведен радиус-вектор . Итак,

что совпадает с векторным моментом силы относительно точки . Вектор , как известно, перпендикулярен плоскости, в которой расположены векторы и , т. е. плоскости треугольника , которой перпендикулярен и векторный момент .

Направление тоже совпадает с направлением . Заметим, что векторный момент силы относительно точки считается вектором, приложенным к этой точке.

Векторный момент силы относительно точки не изменяется от переноса силы вдоль ее линии действия. Он станет равным

нулю, если линия действия силы пройдет через моментную точку.

Рис. 21 

Если сила дана своими проекциями на оси координат и даны координаты точки приложения этой силы (рис. 21), то векторный момент относительно начала координат, согласно формуле (3), после разложения по осям координат вычисляем по формуле

где — единичные векторы, направленные по осям координат.

Используя формулу (4), можно выделить проекции на оси координат:

Модуль векторного момента и косинусы углов его с осями координат определяем по формулам

В формулах (6) числовую величину берем со знаком плюс.

Момент силы относительно оси

Моментом силы относительно оси называют алгебраический момент проекции этой силы на плоскость, перпендикулярную оси, относительно точки пересечения оси с этой плоскостью (рис. 22). Момент силы относительно оси считается положительным, если проекция силы на плоскость, перпендикулярную оси (проекция силы на плоскость является вектором), стремится вращать тело вокруг положительного направления оси против часовой стрелки, и отрицательным, если она стремится вращать тело по часовой стрелке. Момент силы, например, относительно оси обозначим .

 Рис. 22

По определению,   

где — вектор проекции силы на плоскость , перпендикулярную оси , а точка — точка пересечения оси с плоскостью .

Из определения момента силы относительно оси следует, что введенный выше алгебраический момент силы относительно точки можно считать моментом силы относительно оси, проходящей через эту точку, перпендикулярно плоскости, в которой лежат сила и моментная точка. Момент силы относительно оси можно выразить через площадь треугольника, построенного на проекции силы   и точке пересечения оси с плоскостью:

Из формулы (8) можно получить следующие важные свойства момента силы относительно оси:

  1. Момент силы относительно оси равен нулю, если сила параллельна оси. В этом случае равна нулю проекция силы на плоскость, перпендикулярную оси.
  2. Момент силы относительно оси равен нулю, если линия действия силы пересекает эту ось. В этом случае линия действия проекции силы на плоскость, перпендикулярную оси, проходит через точку пересечения оси  с плоскостью и, следовательно, равно нулю плечо силы относительно точки .

В обоих этих случаях ось и сила лежат в одной плоскости. Объединяя их, можно сказать, что момент силы относительно оси равен нулю, если сила и ось лежат в одной плоскости.

Связь момента силы относительно оси с векторным моментом силы относительно точки на оси

Используя формулу (8), имеем (рис. 23)

Векторный момент силы относительно точки , взятой на пересечении оси с перпендикулярной плоскостью , выражается в виде

Векторный момент направлен перпендикулярно плоскости треугольника . Аналогично, для другой точки  оси 

причем векторный момент  направлен перпендикулярно плоскости треугольника . Треугольник является проекцией треугольников и на плоскость . Из геометрии известно, что площадь проекции плоской фигуры равна площади проецируемой фигуры, умноженной на косинус угла между плоскостями, в которых расположены эти фигуры. Угол между плоскостями измеряется углом между перпендикулярами к этим плоскостям. Перпендикуляром к плоскости треугольника является ось , а перпендикулярами к плоскостям треугольников  и —соответственно векторные моменты и . Таким образом, , где — угол между вектором и осью . Отсюда по формулам (8′) и (9) имеем

причем знак полностью определяется знаком .

Аналогично,

т. е.

где  — любая точка на оси .

Формулы (11) и (12) отражают искомую связь между моментом силы относительно оси и векторными моментами силы относительно точек, лежащих на этой оси: момент силы относительно оси равен проекции на эту ось векторного момента силы относительно любой точки на оси.

Эту зависимость между моментом силы относительно оси и векторным моментом силы относительно точки на оси можно принять за определение момента силы относительно оси.

Рис. 23

Формулы для моментов силы относительно осей координат

Используя связь момента силы относительно оси с векторным моментом силы относительно точки на оси, можно получить формулы для вычисления моментов относительно осей координат, если даны проекции силы на оси координат и координаты точки приложения силы. Для оси имеем

Согласно (5),

следовательно,

Аналогично, для осей и

Окончательно

По формулам (13) можно вычислить моменты силы относительно прямоугольных осей координат.

По этим формулам получаются необходимые знаки для  , если проекции силы на оси координат и координаты точки приложения силы подставлять в них со знаками этих величин.

При решении задач момент силы относительно какой-либо оси часто получают, используя его определение, т. е. проецируя силу на плоскость, перпендикулярную оси, и вычисляя затем алгебраический момент этой проекции относительно точки пересечения оси с этой плоскостью.

Момент пары сил. Сложение пар сил. Равновесие пар сил

При изучении теоретической механики необходимо совершенно отчетливо уяснить, что в статике рассматриваются два простейших элемента: сила и пара сил. Любые две силы, кроме сил, образующих пару, всегда можно заменить одной —сложить их (найти равнодействующую). Пара сил нс поддается дальнейшему упрощению, она не имеет равнодействующей и является простейшим элементом.

Действие пары сил на тело характеризуется ее моментом — произведением одной из сил пары на ее плечо (на кратчайшее расстояние между линиями действия сил, образующих пару).

Единицей момента пары сил в Международной системе служит 1 нм (ньютон-метр = 1 н-1ж), а в системе МКГСС (технической)— 1 кГ-м.

Несколько пар сил, действующих на тело в одной плоскости, можно заменить одной парой сил (равнодействующей парой), момент которой равен алгебраической сумме моментов данных пар:

При равновесии пар сил


Если пары сил действуют в одной плоскости, то при решении задач достаточно рассматривать моменты пар как алгебраические величины. Причем знак момента определяется в зависимости от направления вращающего действия пары сил.

Дальнейшее изложение основано на правиле, т. е. считается момент положительным, если пара сил действует против хода часовой стрелки, если же пара сил действует на тело но ходу часовой стрелки, то момент считается отрицательным.

В том случае когда пары сил действуют на тело будучи расположенными в различных плоскостях, гораздо удобнее рассматривать пару сил как вектор, направленный перпендикулярно

к плоскости действия пары сил (рис. 62). Направление вектора в зависимости от направления вращательного действия пары определяется по направлению движения винта с правой нарезкой.

Задача 1.

Определить момент пары сил (рис. 63), если н, АВ — 0,5 м и а = 30°.

Решение.

1.    При определении момента пары сил нужно прежде всего правильно определить плечо пары. При этом необходимо различать следующие понятия: плечо пары сил и расстояние между точками приложения сил нары.

Так как в механике твердого тела сила—скользящий вектор, то действие силы не изменяется при переносе точки ее приложения вдоль линии ее действия. Значит расстояние между точками приложения сил, образующих пару, можно изменять неограниченно. Но плечо пары при этом переносе остается неизменным.

В частном случае расстояние между точками приложения сил, образующих пару, может быть равно плечу.

Чтобы определить плечо данной пары из точки приложения одной из сил, например из точки В, восставим перпендикуляр ВС к линии действия другой силы. Расстояние ВС и есть плечо данной пары сил. Расстояние между точками приложения сил, образующих пару, АВ=0,5 м.

Легко видеть, что


2.    Найдем момент пары сил:

Задача 2.

Как изменится момент пары сил показанной на рис. 64, а (P = 50 н, AВ=0,4 м и а=135), если

повернуть силы так, чтобы они стали перпендикулярными АВ? Решение.

1.    Найдем момент пары при заданном положении ее сил (рис. 64, а).

Из точки В восставим перпендикуляр ВС к линиям действия сил  и найдем его длину:

Момент пары при заданном положении сил


2. Повернем силы из заданного положения на угол =а°— 90э в направлении против хода часовой стрелки (рис. 64, б). При таком положении сил относительно АВ плечом пары сил является расстояние между точками их приложения, поэтому


3.    Сравнивая полученные результаты, видим, что после поворота сил момент пары увеличивается на 20—14,5 = 5,85 н-м.

4.    Легко заметить, что силы могут достичь перпендикулярного положения к АВ после их поворота на угол у в направлении по ходу часовой стрелки (рис. 64, в). В том случае плечом пары является тот же отрезок АВ, но момент пары


Момент пары сил изменяет свой знак.

Задача 3.

К точкам А, С и В, D, образующим вершины квадрата со стороной 0,5 м (рис. 65, а), приложены равные по модулю силы (Р = 12н) таким образом, что они образуют две пары сил

Определить момент равнодействующей пары сил

Решение 1.

Плечи у обеих пар сил равны стороне квадрата поэтому

Решение 2.
1.    Перенесем силы из точек в точки В и D (рис. 65, б). В точках В и D получаются системы сходящихся сил  и одинаковыми модулями.

2.    Сложим попарно эти силы у каждой из точек В и D. В обоих случаях

3.    Силы R, модули которых теперь известны, направлены перпендикулярно к диагонали BD квадрата. Значит эта диагональ является плечом вновь образовавшейся пары сил заменяющей собой две данные.

4.    Найдем момент пары 

и, следовательно,

Эту пару в соответствии со вторым решением можно представить в виде пары с плечом BD (диагональю данного квадрата).

Но можно равнодействующую пару представить и в любом другом виде, например в виде сил Q = 24 и, приложенных к двум любым вершинам квадрата ABCD (рис. 65, в)

Задача 4.

На прямоугольник ABCD (рис. 67) вдоль его длинных сторон действует пара сил Какую пару сил нужно приложить к прямоугольнику, направив силы вдоль его коротких сторон, чтобы уравновесить пару

Решение.

1.    Момент данной пары сил

необходимо уравновесить парой, момент которой обозначим Л1м. Тогда, согласно условию равновесия,

Откуда

2.    Обозначив силы, образующие искомую пару замечая, что ее плечо равно ВС, получим
Отсюда

•Значит к прямоугольнику необходимо приложить пару сил с положительным (направленным против хода часовой стрелки) моментом, равным 48 н м. Силы, образующие эту пару, равняются

20 н каждая и одна из них должна действовать вдоль стороны АВ от А к В, вторая — вдоль стороны CD от С к D.

Задача 5.

Прямолинейный стержень АВ должен находиться в равновесии в положении, показанном на рис. 68, а (угол а = При этом в точках А и В на стержень действуют вертикальные силы образующие пару Какие две равные силы нужно приложить к стержню в точках С и D, направив их перпендикулярно к стержню, чтобы обеспечить равновесие. АВ = 3 м, CD— 1 м,

Решение.

1. Пару сил можно уравновесить только парой сил. Поэтому в точках С и D к стержню необходимо приложить две равные силы так, чтобы они образовали пару сил с моментом, равным моменту пары но имеющим противоположный знак.

Так как пара поворачивает стержень на ходу часовой стрелки, искомые силы должны поворачивать его против хода часовой стрелки (рис. 68, б).

2. Применяем условие равновесия:

Или, подставив значения моментов,
 где

Отсюда

Следовательно, в точках С и D необходимо приложить силы по 150 н каждая, как показано на рис. 68, б.

Момент силы относительно точки

Момент силы относительно точки при решении задач по статике, а затем и по динамике имеет не менее важное значение, чем проекции сил. Поэтому нужно уметь определять эту величину безошибочно. Обычно его числовое значение находят неправильно из-за ошибок, допускаемых при определении плеча.

Чтобы не допускать ошибок при определении моментов сил относительно точки, рекомендуется придерживаться следующего порядка:

  1. Прежде всего нужно научиться «видеть» силу, момент которой определяем, и центр моментов — точку, относительно которой определяем момент (рис. 70 — сила  и центр моментов — точка В).
  2. Затем из центра момента проводим прямую ВЬ перпендикулярно к линии действия силы DF. Длина перпендикуляра ВС от центра момента до линии действия силы и есть плечо.
  3. Потом находим знак момента. При этом если сила стремится повернуть плечо вокруг центра момента против хода часовой стрелки, то считаем момент положительным; если по ходу часовой стрелки, то отрицательным (тоже правило, что и при определении знака момента пары сил).
  4. Находим числовое значение момента силы относительно точки, умножив модуль силы на плечо.

По рис. 70

В частном случае момент силы может равняться нулю. Это происходит тогда, когда центр моментов лежит на линии действия силы, при этом плечо равняется нулю. По рис. 70 момент силы  относительно точки А (или С) равен нулю.

Задача 6.

Определить моменты шести заданных сил (рис. 71) относительно точек А, В и С, если 

Решение 1 — определение моментов гнести заданных сил относительно точки А (рис. 71, а).

1.    Центр моментов в точке А. Через точку А проходят линии действия трех сил Значит для этих сил плечи равны нулю. Следовательно,

2.    Находим момент силы  Опустив из точки А на линию действия

силы перпендикуляр AD, получим плечо силы Длину AD легко найти, так как это катет треугольника ABD:

3.    Величина момента отрицательная (сила поворачивает плечо AD вокруг точки А но ходу часовой стрелки), следовательно,

4.    Находим момент силы Плечом силы является перпендикуляр АЕ к СЕ — линии действия силы Из треугольника АСЕ

Величина момента положительная (плечо АЕ поворачивается около точки А силой против хода часовой стрелки). Следовательно,

5.    Находим момент силы Плечом силы относительно точки А является отрезок АС, так как сила направлена к АС перпендикулярно. Величина момента отрицательная:

Решение 2 — определение моментов сил относительно точки В (рис. 71, б).

1.    Центр моментов в точке В.

2.    Через точку В проходят линии действия двух сил: Следовательно,

3.    Находим момент силы Плечо силы

Величина момента отрицательная:

4.    Находим момент силы Плечо силы

Момент отрицательный:

5.    Находим момент силы Плечо силы

Величина момента положительная:

6.    Находим момент силы Плечом силыявляется отрезок ВС. Момент положительный:

Решение 3 — определение моментов сил относительно точки С (рис. 71, в) рекомендуется выполнить самостоятельно.
 

Ответ.

В задаче  силы расположены так, что либо их плечи определяются очень просто — как катеты прямоугольных треугольников, в которых даны гипотенузы, либо плечи заданы в условии задачи (ВС и АС).

Но иногда некоторые силы заданной системы оказываются расположенными относительно выбранного центра моментов так, что определить длину плеча трудно и требуется, например, предварительно вычислить длины еще одного-двух отрезков. В таких случаях целесообразно силу разложить на две составляющие и применить для определения ее момента теорему Вариньона.

Задача 7.

Определить моменты относительно точки я, приложенных в точках А, В и С, как показано на рис. 72, а. Углы ВС =1,5 м.

Решение.

1.    Относительно точки А моменты сил определяются аналогично

2. Находим момент силы Вариант 1-й (рис. 72, а). Плечо АЕ силы в данном случае определяем из в котором известен только . Значит нужно предварительно определить одну из сторон. Найдем AF:

AF = AB — FB.
Величину FB находим из в котором

следовательно,

И теперь можем определить плечо АЕ:

Раскрываем скобки и заменяем

Момент положительный, следовательно:

Вариант 2-й. Чтобы избежать определения плеча АЕ, которое в данном случае находится после предварительного вычисления двух отрезков (FB и AF), необходимо момент силы относительно точки А найти по теореме Вариньона: момент равнодействующей плоской системы сил относительно любой точки, лежащей в той же плоскости, равен алгебраической сумме моментов составляющих сил относительно той же точки.

Разложим силу на две составляющие: одну, направленную вдоль отрезка ВС, и другую — перпендикулярно к нему (рис. 72, б).

Модуль первой составляющей а ее плечо — отрезок АВ, длина которого задана. Модуль второй составляющей а ее плечо АК = ВС =1,5 м.

Применяя теорему Вариньона, получаем

Как видно, получено точно такое же значение момента, что и в первом варианте решения:

Момент силы относительно точки и оси. Связи между ними — Студопедия

Если к абсолютно твердому телу приложена произвольная пространственная система сил, то изучение ее действия на это тело, в отличие от системы сходящихся сил, требует введения новых понятий. В частности, определения моментов силы относительно точки (центра) и оси. Понятия эти исторически появились в учении Архимеда о равновесии рычагов. Затем были обобщены для любых пространственных систем сил. В начале момент силы относительно точки рассматривался в плоскости, проходящей через линию действия и точку. Он определялся как произведение модуля силы на кратчайшее расстояние (плечо) от линии действия силы до точки, относительно которой брался момент. Причем эта величина считалась положительной, если сила, действующая на тело, стремилась вращать его против движения часовой стрелки и наоборот отрицательной, если она стремилась вращать тело по движению часовой стрелки. Однако при переходе к пространственным системам сил такое определение момента силы относительно точки стало недостаточным. Это в основном было связано с необходимостью учета возможных направлений плоскостей, проходящих через линии действия сил системы и общей для них моментной точки. В связи с этим, вводился такой вектор, приложенный в этой точке, числовое значение которого равно произведению силы на кратчайшее расстояние от моментной точки до линии действия силы и направленный перпендикулярно к той плоскости, в которой расположены линия действия силы и моментная точка. Причем здесь необходимо во внимание принимать и возможный поворот тела под действием силы вокруг выбранного центра. Теперь введем понятие момента силы сформулировав его следующим образом: моментом силы относительно какой-либо точки называется вектор, численно равный произведению модуля силы на плечо и направленный перпендикулярно к плоскости содержащей силу и выбранную точку, таким образом, чтобы с конца этого вектора можно было бы видеть стремление силы вращать тело против движения часовой стрелки.


Так как существуют правая и левая системы координат, то следует конкретно выбрать одну из них, чтобы единым образом определить направление векторного момента силы относительно точки. В дальнейшем будем пользоваться только первой системой координат. Это позволяет применить «правило буравчика», хорошо известное читателям еще со школьной скамьи.

Итак пусть даны сила , приложенная в точке А какого-либо абсолютно твердого тела, и некоторый центр О (рис.2.11). Тогда моментом силы относительно точки О называется вектор, приложенный к центру (или точке) О, направленный перпендикулярно к плоскости треугольника ОАВ в ту сторону, откуда поворот тела, совершаемый силой, виден против хода стрелки часов (по правилу буравчика) и численно равный удвоенный площади треугольника ОАВ, иначе, этот вектор можно представить как векторное произведение радиуса-вектора (т.е вектор, направленный от моментной точки О, к точке А приложения силы и модуль которого равен длине между этими точками) на силу, т.е.


= х . (2.11)

 
 

Рис.2.11.

Здесь для вектора момента силы введено обозначение , где в индексе указывается точка, относительно которой берется момент, а внутри скобки сила, действующая на тело и сверху символа проводится прямая, означающая, что эта величина является векторной. Кроме этого обозначения в существующих литературах по теоретической механике применяются и такие обозначения , .

Теперь докажем, что модуль вектора , представленного формулой (2.11) равен произведению величины силы на плечо, а направление векторного произведения двух векторов и , т.е. х точно совпадает с направлением вектора .

Как известно из векторной алгебры, модуль векторного произведения равен площади параллелограмма, построенного на векторах сомножителях и , т.е.

| × |=rF×sin( ).

Однако из прямоугольного треугольника KOA, где OK=h, имеем rsin( )=h. Следовательно, rFsin( )=F×h=m0( ). Это выражение дает, что модуль вектора равен числовому значению вектора . Кроме того, вектор, равный векторному произведению х направлен по перпендикуляру к плоскости DAOB. Причем в ту сторону, откуда кратчайший поворот вектора к направлению вектора представляется происходящим против хода часовой стрелки, т.е. направление векторного произведения х совпадает с направлением вектора . Таким образом, формула (2.11) полностью определяет модуль и направление момента силы .


ЗАМЕЧАНИЯ

1.При переносе силы по линии ее действия в другие точки, допустим в М1, М2,…., Мn, ее момент относительно данного центра не изменится (рис.2.12).

 
 

Рис.2.12.

То, что направления векторных произведений х , , , …, , перпендикулярных к одной и той же плоскости совпадают, а их числовые значения равны, не вызывают сомнений, так как равны площади соответствующих их параллелограммов, имеющих одно и то же основание и одну и ту же высоту h.

2.Момент силы относительно центра О меняет свой знак при перестановке его сомножителей и . Для доказательства этого составим векторное произведение х . Для этой цели откладываем от произвольной точки В вектор , а из конца его проводим вектор и на отложенных векторах строим параллелограмм (рис.2.13). Тогда направление вектора х противоположно направлению х , а площади параллелограммов между собой равны. Таким образом, х =- х .

 
 

Рис.2.13.

3.Момент силы относительно точки равен нуля, когда ее линия действия проходит через центр (точку) момента, так как в этом случае плечо равно нулю.

4.В случае, когда заданы величина и направление силы и ее момент относительно центра О, то всегда определима линия действия силы. При этом наикратчайшее расстояние h от моментной силы О до линии действия силы , т.е. плечо h находится из выражения h=| | / F.

Момент силы относительно точки О можно также определить, зная проекции силы на прямоугольные декартовы оси и координаты точки приложения силы х, y, z. Действительно, если иметь в виду выражение (2.11), то момент силы относительно этих проекций выражается следующим образом:

= (2.12)

ЗАМЕЧАНИЕ

Если все силы, действующие на тело лежат в одной плоскости, то векторы, изображающие моменты этих сил относительно какой-либо точки, лежащей в той же плоскости, будут перпендикулярны к плоскости. Следовательно, в данном случае моменты сил различаются между собой только числовой величиной и знаком, а не направлением, как в случае пространственной системы сил и их можно рассматривать как скалярные величины. При это величина момента силы равна m0( )=| | = ±F×h, где знак ± зависит от того, куда сила стремиться вращать тело (+ соответствует вращению тела против движения часовой стрелки и наоборот — движения часовой стрелки). Раскрывая определитель (2.12) по столбцам получим:

=(yFz-zFy) +(zFx-xFz) +(xFy-yFx) (2.13)

В то же время, если проекции этого момента силы на прямоугольные декартовы оси координат соответственно обозначим через m( ), m0y( ), m0z( ), то для m0( ) находим:

=m0x( ) +m0y( ) + m0z( ) (2.14)

Далее сравнивая выражения (2.12) и (2.13), которые определяют одну и ту же величину, получим:

m0x( )=yFz-zFy, m0y( )=zFx-xFz, m0z( )=xFy-yFx (2.15)

Формула (2.15) называется аналитическими выражениями моментов силы относительно координатных осей, что будет доказано ниже.

Момент силы относительно оси. Пусть к некоторому твердому телу в точке А приложена сила . Чтобы вычислить момент этой силы относительно какой-либо оси (допустим ею является ось OZ), следует спроектировать силу на перпендикулярную плоскость Pz к указанной оси, затем взять момент от проекции относительно точки О1 пересечения оси с перпендикулярной плоскостью, взяв его со знаком плюс или минус (рис.2.14). Момент силы относительно оси обозначив символом mz( ), согласно выше сказанному, можно его записать следующим образом:

mz( )=±Fn×H (2.16)

При этом момент силы относительно оси mz( ), вычисляемый соотношением (2.16) может быть взят со знаком плюс, если проекция силы стремится вращать тело против движения часовой стрелки; когда смотрим с конца оси и со знаком минус в противном случае.

 
 

Рис.2.14.

ЗАМЕЧАНИЯ

1.Значение момента силы относительно оси может быть также выражено удвоенной площадью треугольника

mz( ) = 2 (2.17)

2.Если проекция силы на перпендикулярную плоскость Pz к оси OZ относительно которой берется момент равна нулю, т.е Fn=0, то линия действия силы параллельна к этой оси.

3.Если расстояние от точки О пересечения оси OZ с перпендикулярной плоскостью Pz до линии действия проекции силы на эту плоскость равно нулю, т.е. H=0, то линия действия силы пересекает ось, относительно которой вычисляется момент.

Таким образом, момент силы относительно некоторой оси равен моменту проекции силы на перпендикулярную плоскость к этой оси, взятому относительно точки пересечения оси с перпендикулярной плоскостью.

Также, можно показать, что проекция вектора момента силы относительно некоторой точки на ось, проходящую через эту точку, равна моменту силы относительно этой же оси. Для этого заметим, что проекция m0z( ) вектора момента на ось OZ равна

m0z( )=прOZ[ ]=| |cosj=2SDOABcosj=±2 (2.18)

так как угол j между вектором и осью OZ равен углу между плоскостями DOAB и Pz, где площадь DOA1B1 есть проекция площади DOAB. Однако по определению момент силы относительно оси определяется выражение вида (2.17). Далее, сравнивая (2.17) с (2.18), имеем, что

m0z( )=mz( ),

что и требовалось показать. Следовательно, чтобы вычислить моменты силы относительно соответствующих осей координат X,Y,Z можно использовать соотношения вида (2.15).

Вопросы для самопроверки

1. Что называется сходящейся системой сил?

2. Как определяется равнодействующая системы сходящихся сил геометрически и аналитический способами?

3. Сформулируйте и запишите условия равновесия системы сходящихся сил в векторной и аналитической формах.

4. Если для плоской системы сходящихся сил åFkx=0, а åFky¹0, что можно сказать о ее равнодействующей?

5. Может ли находиться в равновесии система 3-х сходящихся сил, не лежащих в одной плоскости?

6. Дайте определение векторного момента силы относительно центра. Где он приложен, как направлен?

7. Нарисуйте рисунок, изображающий векторное произведение. Запишите векторный момент силы относительно центра в виде векторного произведения радиуса-вектора точки приложения силы на вектор силы.

8. Как определяется модуль момента силы относительно центра? Что называется плечом силы? Как выражается момент силы относительно центра через площадь треугольника.

9. Дайте определение алгебраического момента силы относительно центра.

10. В каких случаях момент силы относительно центра равен нулю.

11. Дайте определение момента силы относительно оси. В каких случаях этот момент равен нулю.

12. Запишите формулы для моментов силы относительно координатных осей.

13. Какова связь между моментом силы относительно оси и векторным моментом силы относительно любой точки, лежащей на этой оси?


Момент силы относительно точки и оси

Основные понятия статики

Теоретическая механика — наука, изучающая общие законы механического движения и механического взаимодействия материальных тел.

Мех. движением называется перемещение тела по отношению к другому телу, происходящее в пространстве и времени.

Мех. взаимодействием называется такое взаимодействие материальных тел, которое изменяет или стремится изменить характер их мех. движения.

Статика — раздел механики, в котором изучаются методы преобразования систем сил в эквивалентные системы и устанавливаются условия равновесия сил, приложенных к твердому телу.

Материальное тело, размеры которого в рассматриваемых конкретных условиях можно не учитывать, называют материальной точкой (МТ). МТ обладает массой и способностью взаимодействовать с другими телами. Системой МТ или механической системой называется такая совокупность МТ, в которой положение и движение каждой точки зависят от положения и движения других точек этой системы. Тела, расстояния между любыми точками которых остаются неизменными, называют абсолютно твердыми.

Кинематическое состояние тела — состояние покоя или движения определенного характера.

Сила — мера механического взаимодействия тел, определяющая интенсивность и направление этого взаимодействия. Сила определяется тремя элементами: числовым значением (модулем), направлением и точкой приложения. Изображается сила вектором. Прямая, по которой направлена сила называется, называется линией действия силы.

Совокупность нескольких сил, действующих на данное тело, называется системой сил.

Системы сил, под действием каждой из которых твердое тело находится в одинаковом кинематическом состоянии, называются эквивалентными.

Сила, эквивалентная некоторой системе сил, называется равнодействующей.

Сила, равная по модулю равнодействующей и направленная по линии ее действия в противоположную сторону, называется уравновешивающей силой.

Система сил, которая, будучи приложенной к твердому телу, находящемуся в состоянии покоя, не выводящая его из этого состояния, называется системой взаимно уравновешивающихся сил.

Внешними называются силы, действующие на МТ (тела) данной системы со стороны МТ (тел), не принадлежащих этой системе. Внутренними называются силы взаимодействия между МТ (телами) рассматриваемой системы.

Аксиомы статики

1) Аксиома инерции. Под действием взаимно уравновешивающихся сил материальная точка (тело) находится в состоянии покоя или движется равномерно прямолинейно.

2) Аксиома равновесия двух сил. Две силы, приложенные к твердому телу, взаимно уравновешиваются только в том случае, если их модули равны, и они направлены по одной линии прямой в противоположные стороны.

3) Аксиома присоединения и исключения уравновешивающихся сил. Действие системы сил на твердое тело не изменится, если к ней присоединить или из нее исключить систему взаимно уравновешивающихся сил.

Следствие. Не изменяя кинематического состояния абсолютно твердого тела, силу можно переносить вдоль линии ее действия, сохраняя неизменными ее модуль и направление.

4) Аксиома параллелограмма сил. Равнодействующая двух пересекающихся сил приложена в точке их пересечения и изображается диагональю параллелограмма, построенного на этих силах.

φ – угол между направлениями сил

5) Аксиома равенства действия и противодействия. Всякому действию соответствует равное по величине и противоположное по направлению противодействие.

6) Аксиома сохранения равновесия сил, приложенных к деформирующемуся телу при его затвердении. Равновесие сил приложенных к деформирующемуся телу сохраняется при его затвердении.

 

Связи и их реакции

Тело называется свободным, если оно может перемещаться в пространстве в любом направлении.

Тело, ограничивающие свободу движения данного твердого тела, является по отношению к нему связью.

Твердое тело, свобода движения которого ограничена связями, называется несвободным.

Задаваемые силы выражают действие на твердое тело других тел, вызывающих или свободных вызвать изменение его кинематического состояния. Реакцией связи называется сила или система сил, выражающая механическое действие связи на тело.

Принцип освобождаемости твердых тел от связей — несвободное твердое тело можно рассматривать как свободное, на которое кроме задаваемых сил действуют реакции связей.

 

Теорема Пуансо

Приведение силы к заданному центру (метод Пуансо) – силу можно перенести параллельно самой себе в любую точку плоскости, если добавить соответствующую пару сил, момент которой равен моменту этой силы относительно рассматриваемой точки.

Добавим к системе в точке A две силы, равные по величине между собой и величине заданной силы, направленные по одной прямой в противоположные стороны параллельно заданной силе.

 
 

Кинематическое состояние не изменилось (аксиома о присоединении). Исходная сила и одна из добавленных сил противоположно направленная образуют пару сил. Момент этой пары численно равен моменту исходной силы относительно центра приведения. Во многих случаях пару сил удобно изображать дуговой стрелкой.

 
 

 

 

Центр параллельных сил

При сложении двух параллельных сил две параллельные приводятся к одной силе — равнодействующей, линия действия которой направлена параллельно линиям действия сил. Равнодействующая приложена в точке делящей прямую, на расстояния обратно пропорциональные величинам сил.

Поскольку силу можно переносить по линии ее действия, то точка приложения равнодействующей не определена. Если силы повернуть на один и тот же угол и вновь произвести сложение сил, то получим другое направление линии действия равнодействующей. Точка пересечения этих двух линий равнодействующих может рассматриваться как точка приложения равнодействующей, не изменяющая своего положения при повороте всех сил одновременно на один и тот же угол. Такая точка называется центром параллельных сил.

 

Теорема Вариньона

Теорема о моменте равнодействующей силы: момент равнодействующей относительно любой точки равен геометрической сумме моментов составляющих сил относительно этой точки, а момент равнодействующей силы относительно оси равен алгебраической сумме моментов составляющих сил относительно этой оси.

Определим момент равнодействующей силы , приложенной в точке К, относительно произвольно выбранного центра приведения О:

Тогда , что сформулировано в первой части теоремы.

Проекция момента равнодействующей на произвольную ось проходящую через точку O равна:

– угол между осью и направлением .

, что сформулировано во второй части теоремы.

 

Фермы. Методы расчета ферм

Фермой называется геометрически не изменяемая шарнирно-стрежневая конструкция. Если все оси фермы лежат в одной плоскости, то ферму называют плоской. Точки, в которых сходятся оси стержней, называют узлами фермы, а те узлы, которыми ферма опирается на основание, называются опорными узлами. Стержни плоской фермы, расположенные по верхнему контуру, образуют верхний пояс фермы, а расположенные по нижнему контуру — нижний пояс фермы. Вертикальные стержни называются стойками, а наклонные — раскосами.

Способ вырезания узлов. Суть способа заключается в том, что мысленно вырезают каждый узел фермы, прикладывают к нему известные силы и реакции стержней, которые направляют от узла, т.к. неизвестно какие стержни фермы растянуты, а какие сжаты, изначально полагают, что все стержни растянуты, а затем составляются уравнения равновесия сил, приложенных к каждому узлу. Расчет фермы начинают с узлов, к которым приложено не более двух неизвестных сил для плоской фермы, и не более трех для пространственной.

Метод Риттера. Ферму разделяют на две части сечением, проходящим через три стержня, в которых (или в одном из которых) требуется определить усилия, и рассматривают равновесие одной из этих частей. Действие отброшенной части заменяют соответствующими силами, направляя их вдоль разрезанных стержней от узлов, т. е. считая их растянутыми. Затем составляют уравнения моментов относительно точек Риттера (точка пересечения двух из трех перерезанных стержней). Из полученных уравнений определяются усилия в стержнях.

 

Сила трения. Законы трения

Силы трения скольжения появляются при скольжении одного тела по поверхности другого в плоскости соприкосновения тел. Часто приходится учитывать действие этих сил при изучении равновесия тел. С этой целью используются приближенные законы трения, полученные опытным путем:

1. Сила трения возникает лишь тогда, когда приложенные к телу силы стремятся сдвинуть его или оно уже скользит по поверхности другого тела. Сила трения направлена в сторону, противоположную направлению движения или в сторону, противоположную той, в которую приложенные силы стремятся сдвинуть тело.

2. В конкретных условиях сила трения может принимать любые значения в пределах от нуля до некоторого придельного значения , которое достигается в состоянии относительного проскальзывания или в состоянии предельного равновесия тела.

3. Величина предельной силы трения пропорциональна силе нормального давления N между трущимися поверхностями и не зависит от величины площади соприкасания тел:

,

где – коэффициент трения скольжения.

Наибольший угол, на который может отклониться линия действия силы реакции негладкой поверхности от нормали, проведенной к ней в точке контакта тел, называется углом трения скольжения. Тангенс угла скольжения равен коэффициенту трения скольжения:

Трением качения называется сопротивление, возникающие при качении одного тела по поверхности другого. Трение качения возникает оттого, что поверхность катящегося тела и плоскость, по которой тело катится, не абсолютно тверды, а несколько деформируются вследствие давления тела на плоскость.

– вес колеса и его линия действия проходит через центр О катка. Приложим в этой точке горизонтальную силу . В месте контакта катка и поверхности возникает сила трения скольжения , препятствующая проскальзыванию катка.

Под действием силы происходит деформация в месте контакта, в результате чего нормальная реакция смещается в сторону действия силы на некоторое расстояние h. Максимальная величина h=k, соответствующая предельному положению равновесия, называют трением качения.

Значение , соответствующее случаю предельного равновесия,

.

Основные понятия кинематики. Скорость точки. Ускорение

Кинематикой называют раздел механики, в котором движение тел рассматривается без выяснения причин, его вызывающих.

Механическим движением тела называют изменение его положения в пространстве относительно других тел с течением времени.

Тело, относительно которого рассматривается положение изучаемого тела, называется телом отсчета.

Перемещаясь с течением времени из одной точки в другую, тело (материальная точка) описывает некоторую линию, которую называют траекторией движения тела.

Перемещением тела называют направленный отрезок прямой, соединяющий начальное положение тела с его последующим положением. Перемещение есть векторная величина.

Пройденный путь l равен длине дуги траектории, пройденной телом за некоторое время t. Путь – скалярная величина.

Скорость — векторная величина, характеризующая быстроту и направление движения точки в данной системе отсчета.

Для характеристики движения вводится понятие средней скорости:

Мгновенная скорость определяется как предел, к которому стремится средняя скорость на бесконечно малом промежутке времени Δt:

Ускорение — быстрота изменения модуля и направления скорости точки.

Среднее ускорение:

Мгновенное ускорение:

 

Поступательное движение тела. Задание движения. Распределение скоростей и ускорений точек тела

Поступательным движением твердого тела называется такое движение, при котором любая прямая, соединяющая две точки тела, движется параллельно самой себе.

Теорема. Все точки твердого тела, движущегося поступательно, описывают одинаковые (совпадающие при наложении) траектории и в каждый момент времени имеют геометрически равные скорости и ускорения.

Уравнениями поступательного движения твердого тела являются уравнения движения любой точки этого тела — обычно уравнения движения его центра тяжести:

Общие для всех точек твердого тела, движущегося поступательно, скорость ускорение называют скоростью и ускорением поступательного движения твердого тела.

 

Вращательное движение. Задание движения

Вращательным называется такое движение тела, при котором остаются неподвижными все его точки, лежащие на некоторой прямой, называемой осью вращения.

При этом движении все остальные точки тела движутся в плоскостях перпендикулярных оси вращения, и описывают окружности, центры которых лежат на этой оси.

При вращении тела угол поворота изменяется в зависимости от времени: .

Величина, характеризующая быстроту изменения угла поворота с течением времени называется угловой скоростью тела.

Величина, характеризующая быстроту изменения угловой скорости с течением времени, называется угловым ускорением тела.

Уравнение равномерного вращения тела. Вращение тела с постоянной скоростью называется равномерным.

Уравнение равнопеременного вращения тела. Вращение тела, при котором угловое ускорение постоянно, называют равнопеременным вращением.

 

Основные понятия статики

Теоретическая механика — наука, изучающая общие законы механического движения и механического взаимодействия материальных тел.

Мех. движением называется перемещение тела по отношению к другому телу, происходящее в пространстве и времени.

Мех. взаимодействием называется такое взаимодействие материальных тел, которое изменяет или стремится изменить характер их мех. движения.

Статика — раздел механики, в котором изучаются методы преобразования систем сил в эквивалентные системы и устанавливаются условия равновесия сил, приложенных к твердому телу.

Материальное тело, размеры которого в рассматриваемых конкретных условиях можно не учитывать, называют материальной точкой (МТ). МТ обладает массой и способностью взаимодействовать с другими телами. Системой МТ или механической системой называется такая совокупность МТ, в которой положение и движение каждой точки зависят от положения и движения других точек этой системы. Тела, расстояния между любыми точками которых остаются неизменными, называют абсолютно твердыми.

Кинематическое состояние тела — состояние покоя или движения определенного характера.

Сила — мера механического взаимодействия тел, определяющая интенсивность и направление этого взаимодействия. Сила определяется тремя элементами: числовым значением (модулем), направлением и точкой приложения. Изображается сила вектором. Прямая, по которой направлена сила называется, называется линией действия силы.

Совокупность нескольких сил, действующих на данное тело, называется системой сил.

Системы сил, под действием каждой из которых твердое тело находится в одинаковом кинематическом состоянии, называются эквивалентными.

Сила, эквивалентная некоторой системе сил, называется равнодействующей.

Сила, равная по модулю равнодействующей и направленная по линии ее действия в противоположную сторону, называется уравновешивающей силой.

Система сил, которая, будучи приложенной к твердому телу, находящемуся в состоянии покоя, не выводящая его из этого состояния, называется системой взаимно уравновешивающихся сил.

Внешними называются силы, действующие на МТ (тела) данной системы со стороны МТ (тел), не принадлежащих этой системе. Внутренними называются силы взаимодействия между МТ (телами) рассматриваемой системы.

Аксиомы статики

1) Аксиома инерции. Под действием взаимно уравновешивающихся сил материальная точка (тело) находится в состоянии покоя или движется равномерно прямолинейно.

2) Аксиома равновесия двух сил. Две силы, приложенные к твердому телу, взаимно уравновешиваются только в том случае, если их модули равны, и они направлены по одной линии прямой в противоположные стороны.

3) Аксиома присоединения и исключения уравновешивающихся сил. Действие системы сил на твердое тело не изменится, если к ней присоединить или из нее исключить систему взаимно уравновешивающихся сил.

Следствие. Не изменяя кинематического состояния абсолютно твердого тела, силу можно переносить вдоль линии ее действия, сохраняя неизменными ее модуль и направление.

4) Аксиома параллелограмма сил. Равнодействующая двух пересекающихся сил приложена в точке их пересечения и изображается диагональю параллелограмма, построенного на этих силах.

φ – угол между направлениями сил

5) Аксиома равенства действия и противодействия. Всякому действию соответствует равное по величине и противоположное по направлению противодействие.

6) Аксиома сохранения равновесия сил, приложенных к деформирующемуся телу при его затвердении. Равновесие сил приложенных к деформирующемуся телу сохраняется при его затвердении.

 

Связи и их реакции

Тело называется свободным, если оно может перемещаться в пространстве в любом направлении.

Тело, ограничивающие свободу движения данного твердого тела, является по отношению к нему связью.

Твердое тело, свобода движения которого ограничена связями, называется несвободным.

Задаваемые силы выражают действие на твердое тело других тел, вызывающих или свободных вызвать изменение его кинематического состояния. Реакцией связи называется сила или система сил, выражающая механическое действие связи на тело.

Принцип освобождаемости твердых тел от связей — несвободное твердое тело можно рассматривать как свободное, на которое кроме задаваемых сил действуют реакции связей.

 

Момент силы относительно точки и оси

Момент силы — векторная величина, равная произведению радиус-вектора, проведенного от оси вращения к точке приложения силы на вектор этой силы. МС характеризует вращательное действие этой силы на твердое тело.

Алгебраическим моментом силы относительно точки (центра момента) называется взятое со знаком «+» или «-» произведение величины силы на плече силы.

Плечо силы — кратчайшее расстояние от точки центра момента до линии действия силы.

Правило знаков: если сила стремится повернуть тело вокруг точки центра момента против часовой стрелки ставится знак «+», если по часовой знак «-».

Из определения следует, что момент силы относительно точки равен нулю лишь в том случае, когда плече силы равно нулю, т.е. линия действия силы проходит через точку центр момента.

Момент силы относительно оси — момент проекции силы на плоскость перпендикулярную оси взятый относительно точки пересечения оси и плоскости.

Правило знаков: если с острия оси видеть вращение плоскости под действием проекции силы против часовой стрелки ставится знак «+», если по часовой — знак «-».

Из определения следует что момент силы относительно оси равен нулю если:

1) сила пересекает ось (h=0)

2) сила параллельна оси (Fp=0)

3) сила совпадает с осью (h=0 и Fp=0)

Для вычисления момента силы относительно оси необходимо:

1) Выбрать плоскость перпендикулярную оси;

2) Спроецировать силу на эту плоскость;

3) Найти точку пересечения оси и плоскости;

4) Определить плечо относительно точки;

5) Определить знак момента.

 

МОМЕНТ СИЛЫ — это… Что такое МОМЕНТ СИЛЫ?

— величина, характеризующая вращательный эффект силы; имеет размерность произведения длины на силу. Различают момент силы относительно центра (точки) и относительно оси.

M. с. относительно центра О наз. векторная величина M0, равная векторному произведению радиуса-вектора r, проведённого из O в точку приложения силы F, на силу M0 = [rF]или в др. обозначениях M0 = r F (рис.). Численно M. с. равен произведению модуля силы на плечо h, т. е. на длину перпендикуляра, опущенного из О на линию действия силы, или удвоенной площади

треугольника, построенного на центре O и силе:


Направлен вектор M0 перпендикулярно плоскости, проходящей через O и F. Сторона, куда направляется M0, выбирается условно (M0 — аксиальный вектор). При правой системе координат вектор M0 направляют в ту сторону, откуда поворот, совершаемый силой, виден против хода часовой стрелки.

M. с. относительно оси z наз. скалярная величина Mz, равная проекции на ось z вектора M. с. относительно любого центра О, взятого на этой оси; величину Mz можно ещё определять как проекцию на плоскость ху, перпендикулярную оси z, площади треугольника OAB или как момент проекции Fxy силы F на плоскость ху, взятый относительно точки пересечения оси z с этой плоскостью. T. о.,


В двух последних выражениях M. с. считается положительным, когда поворот силы Fxy виден с положит. конца оси z против хода часовой стрелки (в правой системе координат). M. с. относительно координатных осей Oxyz могут также вычисляться по аналитич. ф-лам:


где Fx, Fy, Fz проекции силы F на координатные оси, х, у, z — координаты точки А приложения силы. Величины Mx, My, Mz равны проекциям вектора M0 на координатные оси.

Если система сил имеет равнодействующую, то момент этой равнодействующей относительно любого центра (или оси) равен сумме моментов всех сил системы относительно того же центра (оси) (см. Вариньона теорема). Понятие о M. с. является одним из осн. понятий механики.

Лит. см. при ст. Статика. С. M. Тарг.

Физическая энциклопедия. В 5-ти томах. — М.: Советская энциклопедия. Главный редактор А. М. Прохоров. 1988.

Момент силы относительно точки и оси. Пара сил


⇐ ПредыдущаяСтр 4 из 16Следующая ⇒

1.4.1. Момент силы относительно точки и оси.Рассмотрим силу , приложенную к твердому телу в точке А(x, y, z).

Очевидно, что сила стремится повернуть тело относительно начала координат. Это воздействие назовем моментом силы относительно точки О; оно характеризуется:

— величиной, пропорциональной как модулю силы, так и наименьшему расстоянию от линии действия силы до начала координат (это расстояние на рис.9 обозначено h и называется плечом силы ),

— положением в пространстве плоскости, в которой лежат векторы и (точнее, нормали к этой плоскости),

— направлением вращательного воздействия.

 

Сопоставим воздействию математическую модель в виде связанного с точкой О вектора, равного

 

(5.а)

 

Очевидно, что принятая математическая модель учитывает все указанные выше характеристики моделируемого вращательного воздействия.

Если известны модуль силы и ее плечо относительно точки О, то модуль вектора момента может быть вычислен как:

 

(5.б)

 

Момент силы относительно точки О может быть определен и через проекции соответствующих векторов на оси координатной системы, как

(5.в)

Моментом силы относительно оси называется проекция на эту ось момента силы относительно любой точки оси.

В качестве примера разберем вычисление момента силы относительно оси z.

Сначала (см. рис. 10) разложим силу на две составляющие – вдоль оси z ( ) и перпендикулярную оси z ( ).

 

 

 

Поскольку состояние тела должно сохраняться при замене равнодействующей на ее составляющие, момент от равнодействующей может быть вычислен, как сумма моментов от ее составляющих, т.е.

 

(6)

 

Заметим, что приведенное выше соображение в некоторых источниках носит название теоремы Вариньона.

Очевидно, что составляющая стремится сдвинуть тело вдоль оси z, не стремясь повернуть его вокруг оси z (т.е. ее момент относительно оси z равен нулю). Кратчайшее расстояние от линии действия составляющей до оси z есть h (см. рис. 3.2).

Таким образом,

 

(7)

 

что совпадает с проекцией вектора момента на ось z в формуле (5.в).

Отметим следующие свойства момента силы:

— момент силы относительно точки не меняется при переносе силы вдоль ее линии действия (т.к. не изменяется плечо силы),

— момент силы относительно точки равен нулю, если линия действия силы проходит через эту точку (т.к. плечо силы равно нулю),

— момент силы относительно оси равен нулю, если сила параллельна оси, либо ее пересекает (т.е. если сила и ось лежат в одной плоскости).

 

1.4.2. Пара сил. Момент пары сил.Парой сил называется система двух равных по модулю, параллельных и направленных в противоположные стороны сил , действующих на твердое тело (рис. 11). Кратчайшее расстояние h между линиями действия сил называется плечом пары.

Действие пары сил на твердое тело сводится к некоторому вращательному эффекту, который можно моделировать вектором, называемым моментом пары.

Момент пары рассчитывается как сумма моментов сил, образующих пару, и вычисленных относительно некоторой точки О твердого тела, т.е.:

 

(8)

Так как выбор точки О был произволен, то вектор момента пары можно считать приложенным в любой точке, т.е. это вектор свободный.

Таким образом, математической моделью момента пары сил является свободный вектор, направленный перпендикулярно плоскости действия пары в ту сторону, откуда видно стремление пары повернуть тело против часовой стрелки; модуль вектора пары равен произведению модуля одной из сил на плечо пары (см. рис. 11). Заметим, что связанные векторы момента силы относительно точки В и момента силы относительно точки А совпадают с вектором момента пары по величине и направлению; отмеченное обстоятельство позволяет для расчета момента пары использовать формулы (5).

 

 

Из формулы (8) следует, что две пары сил, имеющих одинаковые моменты, эквивалентны, т.е. оказывают на твердое тело одинаковое механическое действие. При этом пары сил могут располагаться как в одной плоскости, так и в параллельных плоскостях, а величины сил и плеч могут принимать любые значения (при сохранении, конечно, величины их произведения).


Очевидно (см. формулу (8)), что если на тело действует несколько пар с моментами , то сумма моментов всех сил, образующих эти пары, относительно любой точки будет равна

 

(9)

 

то есть вся совокупность этих пар эквивалентна одной паре с моментом . Этот результат выражает теорему о сложении пар.

 


Рекомендуемые страницы:

Момент силы относительно точки. Разложение момента силы по декартовой системе координат

Момент силы относительно произвольной точки.Пусть частица A движется относительно точки О под действием произвольной силы F (см. рис. 9.3).

Моментом силы M относительно произвольной точки О называется векторное произведение радиус-вектора частицы r, проведенного из точки O в точку приложения силы, на вектор силы F:

M = [r, F].

Момент силы относительно оси.

Момент силы относительно оси есть проекция момента силы относительно точки этой оси.

4. Теорема 1:

Моменты силы относительно оси независят от выбора полюса на этой оси.

Вычисление моментов силы относительно оси по этим формулам явл. аналитическим способом.

Теорема 2:

Момент силы оси равен моменту проекции силы на плоскость перпенд. этой оси.

Графоаналитический способ:

a. Спроецировать силу на плоскость параллельную оси.

b. Найти плечо проекции

c. Умножить проекцию на плечо

d. Взять знак +, если выражение силы предусматривается против часовой стрелки, когда ось в глаз.

Характеристики силы. Сила, действующая на твердое тело. Задание силы.

Сила определяется своей величиной, направлением, точкой приложения. Величина силы практически измеряется в весовых единицах (граммах, килограммах, тоннах), направление совпадает с направлением того движения, которое данная сила сообщила бы телу, находившемуся до приложения силы в состоянии покоя. Сила графически изображается вектором – отрезком, начало которого находится в точке приложения силы; указываемое стрелкой направление совпадает с направлением силы, и длина представляет в некотором масштабе величину силы (АВ на рис.). Точка приложения силы, действующей на твердое тело, может быть без изменения действия силы взята где угодно на прямой, совпадающей с направлением силы.

К системе сил, действующих на твёрдое тело, можно всегда добавить две силы, равные по величине, лежащие на одной прямой и направленные в противоположные стороны.

Сходящаяся система сил.

Системой сходящихся силназывается такая система сил, линии действия которой пересекаются в одной точке, которую всегда можно принять за начало координат (рис.5).

Проекции равнодействующей:

;

;

Если , то сила вызывает движение твёрдого тела.

Условие равновесия для сходящейся системы сил:

Понятие о паре сил. Характеристики пары.

Пара сил –это система двух сил, равных по величине и противоположных по направлению.

Момент пары сил –это вектор, направленный перпендикулярно плоскости пары в ту сторону, откуда вращение пары представляется против часовой стрелки.

Объяснитель урока: Момент силы относительно точки в 3D

В этом объяснителе мы узнаем, как найти момент векторных сил. воздействуя на тело около точки в 3D.

Сила или система сил, действующая на тело, может иметь поворотный эффект на этом теле. Этот эффект поворота описывается моментом силы (или системы сил). Возможно, вы уже знакомы с моментом силы, определяемой как скаляр, заданный произведением величины сила с перпендикулярным расстоянием между линией действия сила и точка, относительно которой берется момент: 𝑀 = 𝐹𝑑.⟂

В этом пояснении мы узнаем, что момент силы правильно определяется как вектор.

Момент силы зависит от точки тела, о которой она взят. Направление вектора момента дает направление вращение вокруг той точки, которую создаст сила, в то время как ее величина указывает силу поворачивающего эффекта силы.

Давайте сначала посмотрим, как определяется вектор момента.

Определение: Момент силы

Момент силы ⃑𝐹, действующий на тело, взятая около точки, определяется выражением 𝑀 = ⃑𝑟 × ⃑𝐹, где ⃑𝑟 — вектор положения 𝐴, точка приложения силы ⃑𝐹.

В этом определении мы видим, что система координат была выбрана так, что ее начало отсчета совпадает с точкой, о которой мы говорим. Если бы мы хотели отработать момент силы ⃑𝐹 относительно точки 𝑃 это не начало координат, то мы просто заменим ⃑𝑟 на 𝑃𝐴: 𝑀 = 𝑃𝐴 × ⃑𝐹.

Буква 𝑃 добавлена ​​в качестве индекса к 𝑀 чтобы указать, что момент взят около точки.

Свойства кросс-произведения позволяют нам сначала заключить, что 𝑀 вектор, перпендикулярный плоскости, определяемой и ⃑𝐹.Направление задается формулой Правило правой руки. Это правило иногда объясняют вращением винта: направление вектора ⃑𝐴 × ⃑𝐵 соответствует направлению движения (вверх или вниз) крышки бутылки или гайки, которое можно было бы повернуть в том же направлении вращения, что и при переходе от ⃑𝐴 к ⃑𝐵, как показано на следующей диаграмме.

Эта ссылка на вращение имеет большой смысл, когда речь идет о моменте силы. Давайте возьмем диаграмму, использованную в блоке определений выше, и представим движение стержня, закрепленного в 𝑂 и подвергается единственной силе ⃑𝐹 в 𝐴.Стоит отметить, что для простоты здесь выбрана такая система координат, что И ⃑𝐹 лежат в-плоскости, так что параллельна оси.

Мы видим, что стержень поворачивается по часовой стрелке до совмещения с ⃑𝐹, то есть, когда ⃑𝐹 больше не поворачивает стержень, который действительно описывается тем фактом, что его момент равен нулю (что согласуется с перекрестным произведением двух коллинеарных векторов равны нулю). Это вращение по часовой стрелке соответствует вращению вектора ⃑𝑟 к вектору ⃑𝐹.

Используя правило правой руки, как объяснено выше, это означает, что момент ⃑𝐹 примерно на 𝑂 точек вниз. Другими словами, его единственный компонент, который находится вдоль Ось отрицательна, поскольку положительная ось направлена ​​вверх. Помнить что в трехмерной декартовой системе координат правильный набор единичных векторов ⃑𝑖, ⃑𝑗, ⃑𝑘 такое, что ⃑𝑘 = ⃑𝑖 × ⃑𝑗.

Величина момента определяется выражением ‖‖𝑀‖‖ = ‖‖⃑𝑟‖‖‖‖⃑𝐹‖‖ | 𝜃 |, грех где 𝜃 — угол между ⃑𝑟 и ⃑𝐹.Мы использовали столбцы абсолютных значений вокруг sin𝜃 здесь, потому что если использовать направленные углы, sin𝜃 может быть отрицательным, как в случае вращения по часовой стрелке обсуждалось выше, где 𝜃 — отрицательный угол. Если использовать геометрический углы, то 0≤𝜃≤180∘, и, таким образом, sin𝜃≥0.

Давайте теперь посмотрим, почему такой способ определения величины эквивалент того, что вы, возможно, узнали раньше; а именно, ‖‖𝑀‖‖ = ‖‖⃑𝐹‖‖𝑑⟂, где 𝑑⟂ — перпендикулярное расстояние между линия действия ⃑𝐹 и точка, относительно которой момент взятый.

Взяв наш предыдущий пример стержня, мы можем нарисовать круг с центром 𝐴 и радиус ‖‖⃑𝑟‖‖. Мы можем думать об этом как о единичный круг, масштабированный на ‖‖⃑𝑟‖‖ и повернутый на таким образом, чтобы ось 𝑥, связанная с кругом, была в том же направлении как ⃑𝐹. Скалярная проекция вектора ⃑𝑟 на оси 𝑦, связанной с единичной окружностью, находится отрезок, длина которого тогда равна ‖‖⃑𝑟‖‖ | 𝜃 | sin. Рассматривая теперь отрезок 𝑂𝐶 (заметим, что треугольники 𝐴𝐵𝑌 и 𝑂𝐴𝐶 конгруэнтны), мы видите, что эта длина также является перпендикулярным расстоянием между линией действия ⃑𝐹 и 𝑂.

Чтобы показать, что перпендикулярное расстояние между линией действия ⃑𝐹 и 𝑂 задается формулой ‖‖⃑𝑟‖‖ | 𝜃 | sin, мы также можем использовать то свойство, что два дополнительных углы имеют одинаковое значение синуса ((180 − 𝛼) = 𝛼) sinsin∘. Поскольку ∠𝑂𝐴𝐶 = 180 − 𝜃∘ (здесь 𝜃 — геометрический угол) имеем sinsin∠𝑂𝐴𝐶 = 𝜃. В треугольнике 𝑂𝐴𝐶 имеем 𝑂𝐶 = ‖‖⃑𝑟‖‖∠𝑂𝐴𝐶 = ‖‖⃑𝑟‖‖𝜃.sinsin

Этот результат означает, что для определения величины момента мы можем взять любой угол между линия действия ⃑𝐹 и линия ⃖⃗𝑂𝐴; результат будет таким же.

Как сказано выше, величина момента силы указывает силу силы эффект поворота. Мы можем связать уравнение ‖‖𝑀‖‖ = ‖‖⃑𝐹‖‖𝑑⟂ с тем, что мы все испытали в нашей повседневной жизни, а именно: что поворачивающий эффект данной силы будет увеличиваться, если сила приложена дальше от точки поворота объекта. Это принцип работы рычага.

Метод вычисления векторного произведения двух векторов из его компонентов может быть применен к вычислить момент силы.С помощью силы ⃑𝐹 и вектор положения точки приложения силы ⃑𝐹 определен в системе координат 𝑂, ⃑𝑖, ⃑𝑗, ⃑𝑘 так как ⃑𝐹 = 𝐹⃑𝑖 + 𝐹⃑𝑗 + 𝐹⃑𝑘 и ⃑𝑟 = 𝑟⃑𝑖 + 𝑟⃑𝑗 + 𝑟⃑𝑘, момент ⃑𝐹 относительно точки определяется выражением 𝑀 = ⃑𝑟 × ⃑𝐹 = ||||| ⃑𝑖⃑𝑗⃑𝑘𝑟𝑟𝑟𝐹𝐹𝐹 ||||| = 𝑟𝐹 − 𝑟𝐹⃑𝑖− (𝑟𝐹 − 𝑟𝐹) ⃑𝑗 + 𝑟𝐹 − 𝑟𝐹⃑𝑘. 

Рассмотрим пример, в котором определяется трехмерный вектор момента.

Пример 1: Определение момента силы относительно точки в трех измерениях

Сила величиной 6 Н действует на и представляется вектором на плоскости перпендикулярно оси 𝑦, как показано на рисунке.Определять его вектор момента около в ньютон-сантиметрах.

Ответ

Чтобы ответить на этот вопрос, сначала определим компоненты вектора 𝐴𝐶, который является вектором из точки около где берется момент и точка приложения силы, и сила ⃑𝐹, действующая на 𝐶.

Из рисунка находим 𝐴 (0,0, −16) и 𝐶 (0,16,8), при этом 1 см — единица измерения длина системы координат.Следовательно, 𝐴𝐶 = (0−0) ⃑𝑖 + (16−0) ⃑𝑗 + (8 — (- 16)) ⃑𝑘𝐴𝐶 = 16⃑𝑗 + 24⃑𝑘.cm

Величина силы ⃑𝐹 равна 6 N. Проведем направление ⃑𝐹 в плоскости, перпендикулярной оси, что находится в плоскости, параллельной 𝑥𝑧-плоскости. Остерегайтесь ориентации оси, когда работает в трех измерениях! Мы представили здесь самолет посмотрев на него слева направо на данной диаграмме, то есть такой, что единичный вектор равен направленный вниз по отношению к нашему экрану.

⃑𝐹 лежит в плоскости, перпендикулярной ось; следовательно, 𝐹 = 0. Из рисунка видно, что 𝐹 = ‖‖⃑𝐹‖‖30 = ‖‖⃑𝐹‖‖2, 𝐹 = −‖‖⃑𝐹‖‖30 = −√3‖‖⃑𝐹‖‖2.∘∘sincos

Так как ‖‖⃑𝐹‖‖ = 6, мы находим, что ⃑𝐹 = 3⃑𝑖 − 3√3⃑𝑘.N

Теперь мы можем вычислить момент ⃑𝐹 о 𝐴 как 𝑀 = 𝐴𝐶 × ⃑𝐹 = ||||| ⃑𝑖⃑𝑗⃑𝑘𝐴𝐶𝐴𝐶𝐴𝐶𝐹𝐹𝐹 ||||| 𝑀 = ||||| ⃑𝑖⃑𝑗⃑𝑘0162430−3√3 ||||| 𝑀 = 16⋅ − 3√3− 0⋅24⃑𝑖 − 0⋅ − 3√3 − 3⋅24⃑𝑗 + (0⋅0−3⋅16) ⃑𝑘, 𝑀 =  − 48√3⃑𝑖 + 72⃑𝑗 − 48⃑𝑘⋅. Ncm

Давайте теперь посмотрим на пример, где несколько сил действуют в точке, создавая момент.

Пример 2: Нахождение вектора результирующего момента двух сил относительно начала координат в трех измерениях

На рисунке, если силы ⃑𝐹 = −7⃑𝑖 − ⃑𝑗 + 3⃑𝑘 и ⃑𝐹 = −7⃑𝑖 + 8⃑𝑗 − 6⃑𝑘 действуют на точку 𝐴, где ⃑𝐹 и ⃑𝐹 измеряются в ньютонах, определить вектор момента равнодействующей относительно точки 𝑂 в ньютон-сантиметрах.

Ответ

Две силы действуют в 𝐴, ⃑𝐹 и ⃑𝐹.Поскольку две силы действуют в одной точке, сумма их моментов равна моменту их равнодействующей. По этой причине, нас просят найти момент их равнодействующей (т. е. их суммы).

Начнем с нахождения равнодействующей ⃑𝐹 ⃑𝐹 и ⃑𝐹: ⃑𝐹 = ⃑𝐹 + ⃑𝐹⃑𝐹 = −7⃑𝑖 − ⃑𝑗 + 3⃑𝑘 − 7⃑𝑖 + 8⃑𝑗 − 6⃑𝑘⃑𝐹 = −14⃑𝑖 + 7⃑𝑗 − 3⃑𝑘.

Мы хотим вычислить момент ⃑𝐹 относительно точки 𝑂, начало координат, поэтому нам нужно найти вектор положения 𝐴, ⃑𝑟.Из рисунка находим что ⃑𝑟 = 9⃑𝑖 + 12⃑𝑗 + 8⃑𝑘.

Момент ⃑𝐹 около точки 𝑂 дан кем-то 𝑀 = ⃑𝑟 × ⃑𝐹 = ||||| ⃑𝑖⃑𝑗⃑𝑘𝑟𝑟𝑟𝐹𝐹𝐹 ||||| 𝑀 = |||| ⃑𝑖⃑𝑗⃑𝑘9128−147−3 |||| 𝑀 = (12⋅ (−3) −7⋅8) ⃑𝑖 — (9⋅ (−3) — (- 14) ⋅8) ⃑𝑗 + (9⋅7 — (- 14) ⋅12) ⃑𝑘𝑀 =  − 92⃑𝑖 − 85⃑𝑗 + 231⃑𝑘⋅. Ncm

Теперь рассмотрим пример, в котором определяются неизвестные компоненты вектора силы. с момента около точки из-за силы.

Пример 3: Нахождение неизвестных компонентов силы по ее вектору положения и моменту Компоненты вокруг оси в трех измерениях

Если сила ⃑𝐹 = 𝑚⃑𝑖 + 𝑛⃑𝑗 − ⃑𝑘 действует в точке, вектор положения которой равен ⃑𝑟 = 14⃑𝑖 − ⃑𝑗 + 12⃑𝑘 и 𝑥- и 𝑦-компоненты момента силы ⃑𝐹 относительно исходной точки соответственно 73 и 242 единицы момента, найти значения 𝑚 и 𝑛.

Ответ

Поскольку здесь приведены 𝑥- и 𝑦-компоненты момент силы ⃑𝐹 относительно начала координат, сначала поработаем момент, используя векторы положения и силы. Это даст нам три компонента момента через 𝑚 и 𝑛: 𝑀 = ⃑𝑟 × ⃑𝐹 = ||||| ⃑𝑖⃑𝑗⃑𝑘𝑟𝑟𝑟𝐹𝐹𝐹 ||||| 𝑀 = |||| ⃑𝑖⃑𝑗⃑𝑘14−112𝑚𝑛 − 1 |||| 𝑀 = ((- 1) ⋅ (−1) −12𝑛) ⃑𝑖− (14⋅ (−1) −12𝑚) ⃑𝑗 + (14𝑛 + 𝑚) ⃑𝑘𝑀 = (1−12𝑛) ⃑𝑖 + (14 + 12𝑚) ⃑𝑗 + (14𝑛 + 𝑚) ⃑𝑘.

Приравнивание 𝑥- и 𝑦-составляющих 𝑀 со значениями, указанными в вопросе, дает

1−12𝑛 = 73,14 + 12𝑚 = 242. (1) (2)

Решим уравнение (1): −12𝑛 = 72, 𝑛 = 72−12 = −6.

И уравнение (2) дает 12𝑚 = 228, 𝑚 = 22812 = 19.

Ответ: 𝑚 = 19 и 𝑛 = −6.

Давайте теперь посмотрим на другой пример, в котором нас просят определить неизвестные компоненты вектора положения точки, в которой действует сила, используя момент силы.

Пример 4: Нахождение неизвестных координат точки, лежащей на линии действия силы с учетом ее вектора и момента силы относительно точки в трех измерениях

Момент силы ⃑𝐹 относительно начала координат равен 𝑀, где ⃑𝐹 = ⃑𝑖 − 2⃑𝑗 − ⃑𝑘 и 𝑀 = 20⃑𝑖 + 27⃑𝑗 − 34⃑𝑘.Учитывая, что сила проходит через точку, 𝑦-координата которой равна 4, найти -и 𝑧-координаты точки.

Ответ

Момент силы, действующей в точке 𝐴 относительно начала координат, равен задается перекрестным произведением вектора положения, ⃑𝑟, а сила ⃑𝐹: 𝑀 = ⃑𝑟 × ⃑𝐹.

В этом вопросе нам не дается точка приложения силы, но нас просят найти 𝑥-координату точки, лежащей на линия действия силы.

Однако мы знаем, что ⃑𝑟 × ⃑𝐹 = ‖‖⃑𝑟‖‖‖‖⃑𝐹‖‖ | 𝜃 | ⃑𝑛, грех где 𝜃 — угол между ⃑𝑟 и ⃑𝐹 и ⃑𝑛 единичный вектор, перпендикулярный плоскости, определяемой и ⃑𝐹 (при условии ⃑𝑟 и ⃑𝐹 неколлинеарны). В качестве ‖‖⃑𝑟‖‖ | 𝜃 | sin равно перпендикулярному расстоянию между линией действия и точкой, о которой момент взят (здесь начало координат),, мы можем написать 𝑀 = ‖‖⃑𝐹‖‖𝑑⃑𝑛.⟂

Мы видим, что если взять ⃑𝑟 как вектор положения любой точки на линии действия ⃑𝐹 даст тот же момент, 𝑀.

Таким образом, теперь мы можем вычислить 𝑀, вычислив ⃑𝑟 × ⃑𝐹, где ⃑𝑟 является вектором положения. точки на линии действия с координатами (𝑥, 4, 𝑧): 𝑀 = ⃑𝑟 × ⃑𝐹 = ||||| ⃑𝑖⃑𝑗⃑𝑘𝑟𝑟𝑟𝐹𝐹𝐹 ||||| 𝑀 = |||| ⃑𝑖⃑𝑗⃑𝑘𝑥4𝑧1−2−1 |||| 𝑀 = ((- 1) ⋅4 — (- 2) 𝑧 ) ⃑𝑖 — ((- 1) 𝑥 − 1⋅𝑧) ⃑𝑗 + ((- 2) ⋅𝑥 − 1⋅4) ⃑𝑘𝑀 = (- 4 + 2𝑧) ⃑𝑖 + (𝑥 + 𝑧) ⃑𝑗 + (- 2𝑥 −4) ⃑𝑘.

В вопросе говорится, что 𝑀 = 20⃑𝑖 + 27⃑𝑗 − 34⃑𝑘.

Приравнивание обоих выражений для 𝑀 дает уравнение для каждого компонента: −4 + 2𝑧 = 20, 𝑥 + 𝑧 = 27, −2𝑥 − 4 = −34.

Первое уравнение можно решить следующим образом: 2𝑧 = 24𝑧 = 12.

И третье уравнение дает значение 𝑥: 𝑥 = −34 + 4−2𝑥 = 15.

Теперь мы можем проверить, что эти решения подтверждают второе уравнение, чтобы сделать убедитесь, что наша система уравнений непротиворечива: 𝑥 + 𝑧 = 15 + 12 = 27.

Сила проходит через точку с координатами (15,4,12).

Давайте посмотрим на пример, в котором перпендикулярное расстояние между линией действия силы и точки.

Пример 5: Нахождение вектора момента силы, действующей на заданную точку, и длины Перпендикуляр между начальной точкой и линией действия силы

Найдите момент 𝑀 силы ⃑𝐹 об исходной точке, учитывая, что ⃑𝐹 = −2⃑𝑖 + ⃑𝑗 + ⃑𝑘 и действует в точке 𝐴 вектор положения которого равен ⃑𝑟 = 6⃑𝑖 + 6⃑𝑗 − 3⃑𝑘 относительно исходной точки, затем определить длину 𝐿 перпендикулярный сегмент, проведенный от исходной точки до линии действия силы ⃑𝐹.

Ответ

Нас просят сначала найти момент силы ⃑𝐹 о происхождении. Это дается 𝑀 = ⃑𝑟 × ⃑𝐹 = ||||| ⃑𝑖⃑𝑗⃑𝑘𝑟𝑟𝑟𝐹𝐹𝐹 ||||| 𝑀 = |||| ⃑𝑖⃑𝑗⃑𝑘66−3−211 |||| 𝑀 = (6⋅1−1⋅ (−3)) ⃑𝑖 — (6⋅1 — (- 2) ⋅ (−3)) ⃑𝑗 + (6⋅1 — (- 2) ⋅6) ⃑𝑘𝑀 = 9⃑𝑖 + 18⃑𝑘.

Тогда мы попросили определить длину 𝐿 перпендикуляра отрезок, проведенный от исходной точки до линии действия силы ⃑𝐹. 𝐿 это то, что обычно называют to как перпендикулярное расстояние между началом координат и линией действия ⃑𝐹, обозначаемый 𝑑⟂.

Зная, что ‖‖𝑀‖‖ = ‖‖⃑𝑟‖‖‖‖⃑𝐹‖‖ | 𝜃 | = ‖‖⃑𝐹‖‖𝑑, sin⟂ мы видим, что, поскольку 𝐿 = 𝑑⟂, задается формулой 𝐿 = ‖‖𝑀‖‖‖‖⃑𝐹‖‖.

У нас есть ‖‖𝑀‖‖ = 𝑀 + 𝑀 + 𝑀‖‖𝑀‖‖ = √9 + 18‖‖𝑀‖‖ = √9 + 9⋅2‖‖𝑀‖‖ = √9⋅5‖‖ 𝑀‖‖ = 9√5 а также ‖‖⃑𝐹‖‖ = 𝐹 + 𝐹 + 𝐹‖‖⃑𝐹‖‖ = √2 + 1 + 1‖‖⃑𝐹‖‖ = √6.

Следовательно, 𝐿 = ‖‖𝑀‖‖‖‖⃑𝐹‖‖𝐿 = 9√5√6𝐿 = 9√306𝐿 = 3√302.lengthunits

Давайте подведем итог тому, что мы узнали в этом объяснении.

Ключевые моменты

  • Момент силы вокруг точки описывает эффект поворота вокруг это точка силы.
  • Момент силы ⃑𝐹, действующий на тело, взято вокруг точки, начала системы координат, дан кем-то 𝑀 = ⃑𝑟 × ⃑𝐹 = ||||| ⃑𝑖⃑𝑗⃑𝑘𝑟𝑟𝑟𝐹𝐹𝐹 |||||,  где ⃑𝑟 — вектор положения, точка приложения силы ⃑𝐹.
    Когда момент силы ⃑𝐹 берется относительно точки 𝑃, который не является источником, ⃑𝑟 в приведенное выше уравнение заменяется на.
  • Учитывая свойства векторного произведения, величина момента определяется выражением ‖‖𝑀‖‖ = ‖‖⃑𝑟‖‖‖‖⃑𝐹‖‖ | 𝜃 |, грех где 𝜃 — угол между ⃑𝑟 и ⃑𝐹.
  • Поскольку перпендикулярное расстояние 𝑑⟂ между линией действия силы ⃑𝐹 и точка, относительно которой измеряется момент, равна равно ‖‖⃑𝑟‖‖ | 𝜃 | sin (или ‖‖𝐴𝑃‖‖ | 𝜃 | грех), у нас есть ‖‖𝑀‖‖ = ‖‖⃑𝐹‖‖𝑑; ⟂ то есть, 𝑑 = ‖‖𝑀‖‖‖‖⃑𝐹‖‖.⟂

Момент силы, Пара

Момент силы, Пара
SolitaryRoad.com
Владелец сайта: Джеймс Миллер
 

[ Дом ] [ Вверх ] [ Информация ] [ Почта ]

Момент силы, пара

Момент силы (или крутящего момента) вокруг оси.На рис. 1 тело, которое свободно вращаться вокруг оси AB, действует сила F. Мы хотим найти момент F относительно оси AB. На рис. 2 — поперечный разрез S взят перпендикулярно оси вращения AB с буквой O, обозначающей ось. Усилие F s на рисунке изображена составляющая F лежащей в плоскости поперечного сечения S (F — разлагается на составляющую, параллельную плоскости и компонента, перпендикулярного плоскости самолет). Расстояние d от оси O до линия действия F s называется силовым рычагом или момент руки.

Момент силы (или крутящего момента) вокруг оси. Момент силы F относительно оси AB определяется как произведение плеча момента и силы F s , т. Е.


Момент F = d × F с

Момент силы вокруг оси является мерой эффективность силы при вращении вокруг ось. Момент часто считается вектором количество. В этом случае момент F определяется как

млн = × F с

где вектор от оси O к точке C в Инжир.2. Это вектор, перпендикулярный плоскости S с направление, заданное правилом правой руки для креста продукты.

Момент силы относительно точки. Момент силы F относительно точки O — это векторная величина r × F, где r — вектор положения из центр момента O в любую точку на линии действия силы F. См. рис. 3. Его величина равна произведение величины F на перпендикулярное расстояние от точки O до линии действия F. направление перпендикулярно плоскости, определяемой буквой O и линией действия F.

Обозначим момент F около O через M O (F) и пусть

1) r = xi + yj + zk, F = F x i + F y j + F z k, M O (F) = M x i + M y j + M z k


Тогда по определению

Раскрывая определитель, получаем

M O (F) = i (F z y — F y z) + j (F x z — F z x) + k (F y x — F x y)

Сравнение этого выражения с выражением для M O (F) в 1) выше, у нас есть

M x = F z y — F y z, M y = F x z — F z x, M z = F y x — F x y

Можно показать, что M x , M y, и M z — соответствующие моменты силы F относительно x, оси y и z, как определено в Def.1 выше (момент силы вокруг оси). Это факт, что делает момент силы вокруг точки M = r × F очень полезным.

Def. Центр моментов. Дело в том, какие моменты вычислены. Обозначая момент как r × F, момент центр — это исходная точка, из которой вектор положения r к любому точка на линии действия силы F.

По умолчаниюПара. Пара — это пара параллельных сил, равных по по величине и противоположному направлению. См. Рис. 4. Два таких силы не могут быть объединены в одну силу, так как их сумма во всех направлениях равно нулю. Их единственный эффект — эффект создающий крутящий момент, склонность к вращению.

Момент пары. Момент пары о некоторый центр момента задается суммой моментов двух сил, составляющих пара о тот момент центр. На рис. 4 изображена пара, состоящая из двух сил в плоскости.Момент пары относительно точки О (ось вращения, перпендикулярная плоскости) задан по

M O = (a + d) F — aF

или

M O = Fd.

Чтобы найти момент пары в трехмерном пространстве, мы можем использовать концепцию момент о точке. Рассмотрим рис. 5. момент C пары, показанной там, состоящей из две противоположные параллельные силы F, разделенные расстояние d равно

C = M o (F) + M o (-F) = r 1 × F + r 2 × -F = (r 1 — r 2 ) × F

или

C = a × F

, где a = r 1 — r 2

По определению перекрестного произведения величина C равна | a × F | = aFsin θ и sin θ равен d,

перпендикулярное расстояние между силами.

Момент M пары — это вектор, перпендикулярный плоскости двух сил. Его величина — Fd, где F — величина сил, а d — расстояние по перпендикуляру. между ними. Его направление определяется условием для правой руки: согните пальцы правой руки. руку в том направлении, в котором пара стремится вызвать вращение, а большой палец указывает на направление пары.

Пары подчиняются всем законам векторов. Размер пары одинаков для всех моментов центры.

Разрешение силы на силу и пару. Рассмотрим тело с силой F, действующей в точке А, как показано на рис. 6а. В любой другой точке B две равные и противоположные силы F могут быть применяется без внешних воздействие на тело как показано на рис. 6б. Если эти силы параллельно исходной силе F, то пара M = Fd является образованный исходным F и сила F в противоположном направление в B. На рис. 6c мы увидеть, как исходная сила F в точке A была заменена силой той же величины и направления в точке B и пара.Величина пары равна Fd, произведению величины F и расстояния. через которую была изменена его линия действий. Силу всегда можно заменить равной сила, имеющая любую параллельную линию действия и соответствующую пару. Теперь мы заявляем об этом с помощью векторные обозначения в следующей теореме:

Теорема 1. Единственная сила F, действующая в точке A, может быть заменена равной и аналогичным образом направленная сила, действующая в любой другой точке B, и пара C = r × F, где r — вектор из B в A.

Отсюда следует, что данная сила и пара, которые действуют в одной плоскости, могут быть объединены, чтобы дать единая равная сила, имеющая уникальное направление действия.

Список литературы.

Маклин, Нельсон. Инженерная механика. (Шаум)

Meriam. Механика, Часть I — Статика.

Ещё с сайта SolitaryRoad.com:

Путь истины и жизни

Божье послание миру

Иисус Христос и Его учение

Мудрые слова

Путь просветления, мудрости и понимания

Путь истинного христианства

Америка, коррумпированная, развратная, бессовестная страна

О целостности и ее отсутствии

Проверка на христианство человека — это то, что он есть

Кто попадет в рай?

Высший человек

О вере и делах

Девяносто пять процентов проблем, с которыми сталкивается большинство людей. пришли из личной глупости

Либерализм, социализм и современное государство всеобщего благосостояния

Желание причинить вред, мотивация поведения

Обучение таково:

О современном интеллектуализме

О гомосексуализме

О самодостаточной загородной жизни, усадьбе

Принципы жизни

Актуальные притчи, заповеди, аранжировка Котировки.Общие поговорки. Альманах бедного Ричарда.

Америка сбилась с пути

Действительно большие грехи

Теория формирования характера

Моральное извращение

Ты то, что ты ешь

Люди подобны радиотюнерам — они выбирают и слушайте одну длину волны и игнорируйте остальные

Причина черт характера — по Аристотелю

Эти вещи идут вместе

Телевидение

Мы то, что мы едим — живем в рамках диеты

Как избежать проблем и неприятностей в жизни

Роль привычки в формировании характера

Истинный христианин

Что такое истинное христианство?

Личные качества истинного христианина

Что определяет характер человека?

Любовь к Богу и любовь к добродетели тесно связаны

Прогулка по пустынной дороге

Интеллектуальное неравенство между людьми и властью в хороших привычках

Инструменты сатаны.Тактика и уловки, используемые дьяволом.

Об ответе на ошибки

Настоящая христианская вера

Естественный путь — Неестественный путь

Мудрость, разум и добродетель тесно связаны

Знание — это одно, мудрость — другое

Мои взгляды на христианство в Америке

Самое главное в жизни — понимание

Оценка людей

Мы все примеры — хорошо или плохо

Телевидение — духовный яд

Главный двигатель, который решает, «кто мы»

Откуда берутся наши взгляды, взгляды и ценности?

Грех — серьезное дело.Наказание за это настоящее. Ад реален.

Самостоятельная дисциплина и регламентация

Достижение счастья в жизни — вопрос правильных стратегий

Самодисциплина

Самоконтроль, сдержанность, самодисциплина — основа всего в жизни

Мы наши привычки

Что создает моральный облик?


[ Дом ] [ Вверх ] [ Информация ] [ Почта ]

Численность, математика и статистика — набор академических навыков

Моменты (Механика)

Моменты

Поворачивающий эффект силы на тело, на которое она действует, измеряется моментом силы.

Момент силы зависит от величины силы и расстояния от оси вращения.

Момент силы относительно точки равен (величине силы) $ \ times $ (перпендикулярное расстояние линии действия силы от точки).

Когда на тело действует несколько сил, моменты вокруг точки могут быть добавлены, если задано положительное направление (по или против часовой стрелки) и учитывается для каждого момента.

Рабочий пример: Нахождение момента
Найдите момент $ F $ около $ P $

Найдите момент $ F $ относительно $ P $ на диаграмме выше.{\ circ}), \\ & = 64.241 \ mathrm {Nm} \ text {(до 3d.p.).} \ end {align} Момент силы измеряется в ньютон-метрах $ \ mathrm {N m } $, поэтому момент $ F $ относительно $ P $ равен $ 64.241 \ mathrm {Nm} $.

Сумма моментов

На схеме показан набор сил, действующих на световой стержень. Вычислите сумму моментов относительно точки $ P $.

Решение

Каждая сила уже перпендикулярна точке $ P $.

Момент силы $ 6 \ mathrm {N} $ равен $ 6 \ times 2 = 12 \ mathrm {N m} $ против часовой стрелки.

Момент силы $ 14 \ mathrm {N} $ равен $ 14 \ times 2 = 28 \ mathrm {N m} $ по часовой стрелке.

Момент силы $ 5 \ mathrm {N} $ равен $ 5 \ times (2 + 3) = 25 \ mathrm {Nm} $ против часовой стрелки.

Итого по часовой стрелке $ = 28 \ mathrm {N m} $ и всего против часовой стрелки $ = 37 \ mathrm {Nm} $. Следовательно, сумма моментов равна $ 37 — 28 = 9 \ mathrm {Nm} $ против часовой стрелки. Поскольку общая сумма против часовой стрелки была больше, мы выбрали против часовой стрелки в качестве положительного направления.

Рабочий пример: определение момента, когда заданное расстояние не перпендикулярно
Найдите момент силы около $ P $

Найдите момент силы около $ P $ на диаграмме выше.{\ circ}), \\ & = 66.684 \ mathrm {Nm} \ text {против часовой стрелки (3 d.p.).} \ end {align}

Рабочий пример: тела, находящиеся в состоянии равновесия
Поиск нормальных реакций

Предположим, что существует равномерный стержень длины $ 9 \ mathrm {m} $ и веса $ 30 \ mathrm {N} $. Он опирается на опору $ X $ в одной конечной точке и опору $ Y $, которая находится на расстоянии $ 5 \ mathrm {m} $ от опоры $ X $. Рассчитайте величину реакции на каждой из опор.

Решение

Мы можем нарисовать диаграмму, показывающую все действующие силы.

Вес стержня воздействует на его центр масс — поскольку это однородный стержень, вес действует в средней точке. Стержень находится в равновесии, поэтому общие силы, действующие вверх, будут равны полным силам, действующим вниз, когда мы решаем вертикально. \ begin {уравнение} R_X + R_Y = 30. \ end {уравнение} Рассмотрим моменты относительно точки $ X $, у нас есть, что моменты по часовой стрелке будут равны моментам против часовой стрелки, поэтому \ begin {align} 30 \ times 4.5 & = R_Y \ times ( 4,5 + 0.5), \\ 135 & = 5R_Y, \\ 27 \ mathrm {N} & = R_Y. \ end {align} Теперь мы можем использовать это значение, чтобы найти $ R_X $ \ begin {align} R_X + R_Y & = 30, \\ R _ X & = 30 — 27, \\ & = 3 \ mathrm {N}. \ end {align} Следовательно, реакция в точке $ X $ равна $ 3 \ mathrm {N} $, а реакция в точке $ Y $ равна $ 27 \ mathrm {N} $.

Нахождение центра масс в неоднородном теле

Предположим, что существует неоднородный стержень $ AB $ длины $ 10 \ mathrm {m} $ и веса $ 15 \ mathrm {N} $. Он находится в горизонтальном положении и опирается на опоры в точках $ C $ и $ D $.Расстояние $ A C = 3m $ и $ AD = 7m $. Величина реакции при $ C $ в четыре раза превышает величину реакции при $ D $. Найдите расстояние от центра масс стержня до $ A $.

Решение

Мы можем нарисовать диаграмму, показывающую все действующие силы.

Здесь мы предположили, что центр масс действует в точке $ x $ m из точки $ A $. Поскольку стержень неоднороден, нельзя сказать, что центр масс находится в средней точке стержня.Разрешаем по вертикали \ begin {align} 4R + R & = 15, \\ 5R & = 15, \\ R & = 3. \ end {align} Теперь берем моменты около точки $ A $ \ begin {align} 15 \ раз x & = (4R \ times 3) + (R \ times 7), \\ & = 36 + 21, \\ & = 57, \\ x & = 3.8. \ end {align} Центр масс находится в $ 3.8 \ mathrm {m} $ от $ A $.

Момент силы — проблемы и решения

1. Если F R — это чистая сила F 1 , F 2 и F 3 , какова величина силы F 2 и x?

Известный:

Сила нетто (F R ) = 40 Н

Усилие 1 (F 1 ) = 10 Н

Усилие (F 3 ) = 20 Н

Разыскивается: Величина силы F 2 и расстояние x

Решение:

Найти величину силы F 2 :

Принудительно указывает направленность вверх, отрицательный знак со знаком, и направленный вниз, отрицательный знак.

ΣF = 0

— Факс R + Факс 1 + Факс 2 — Факс 3 = 0

— 40 + 10 + Ф 2 — 20 = 0

— 30 + Факс 2 — 20 = 0

— 50 + Факс 2 = 0

F 2 = 50 Ньютон.

Знак плюс означает, что сила направлена ​​вверх.

Найдите x.

Выберите A в качестве оси вращения.

τ 1 = F 1 l 1 = (10 Н) (1 м) = 10 Нм

Крутящий момент 1 вращает балку против часовой стрелки, поэтому мы назначаем положительный знак крутящему моменту 3.

τ 2 = F 2 x = (50) (x) = 50x Нм

Крутящий момент 1 вращает балку против часовой стрелки, поэтому мы назначаем положительный знак крутящему моменту 3.

τ 3 = F 3 x = (20 Н) (1,75 м) = -35 Нм

Крутящий момент 2 вращает балку по часовой стрелке, поэтому мы назначаем отрицательный знак крутящему моменту 2.

Чистый момент силы:

Στ = 0

10 + 50x — 35 = 0

50x — 25 = 0

50x = 25

х = 25/50

х = 0.5 м

2. Силы F 1 , F 2 , F 3 и F 4 действуют на стержень ABCD, как показано на рисунке. Если не учитывать массу стержня, какова величина момента силы относительно точки A.

Ось вращения = точки A.

Известный:

Усилие F 1 = 10 Н, плечо рычага l 1 = 0

Усилие F 2 = 4 Н, плечо рычага l 2 = 2 метра

Усилие F 3 = 5 Н, плечо рычага l 3 = 3 метра

Усилие F 4 = 10 Н, плечо рычага l 4 = 6 метров

Разыскивается: момент силы около точки A

Решение:

Момент силы 1 (τ 1 ) = F 1 l 1 = (10) (0) = 0

Момент силы 2 (τ 2 ) = F 2 l 2 = (4) (2) = -8 Нм

Момент силы 3 (τ 3 ) = F 3 l 3 = (5) (3) = 15 Нм

Момент силы 4 (τ 4 ) = F 4 л 4 = (10) (6) = -60 Нм

Если крутящий момент вращает шток против часовой стрелки, то присваивается положительный знак.

Если крутящий момент вращает шток по часовой стрелке, то мы присваиваем отрицательный знак.

Равнодействующая момента силы:

τ = 0-8 Нм + 15 Нм — 60 Нм

τ = -68 Нм + 15 Нм

τ = -53 Нм

Знак минус указывает на то, что момент силы вращает шток по часовой стрелке.

3. На стержень действуют три силы: F A = F C = 10 Н и F B = 20 Н, как показано на рисунке ниже. Если расстояние AB = BC = 20 см, каков момент силы относительно точки C.

Известный:

Вращение оси в точке C.

Расстояние между F A и осью вращения (r AC ) = 40 см = 0,4 метра

Расстояние между F B и осью вращения (r BC ) = 20 см = 0,2 метра

Расстояние между F C и осью вращения (r CC ) = 0 см

F A = 10 Ньютон

F B = 20 Ньютон

F C = 10 Ньютон

Разыскивается: Равнодействующая момента силы относительно точки C.

Решение:

Момент силы A:

Στ A = (F A ) (r AC sin 90 o ) = (10 Н) (0,4 м) (1) = -4 Н · м

Знак минус означает, что момент силы вращает шток по часовой стрелке.

Момент силы B:

Στ B = (F B ) (r BC sin 90 o ) = (20 Н) (0,2 м) (1) = 4 Н · м

Знак плюс означает, что момент силы вращает шток против часовой стрелки.

Момент силы C:

Στ C = (F C ) (r CC sin 90 o ) = (10 Н) (0) (1) = 0

равнодействующая момента силы:

Στ = Στ 1 + Στ 2 + Στ 3

Στ = -4 + 4 + 0

Στ = 0 Нм

4. Длина лозы 50см. На стержень действуют три силы, как показано на рисунке ниже. Если осью вращения является точка C, то каков будет чистый момент силы.

Известный:

Вращение оси в точке C.

Расстояние между F 1 и осью вращения составляет (r 1 ) = 30 см = 0,3 метра

Расстояние между F 2 и осью вращения (r 2 ) = 10 см = 0,1 метра

Расстояние между F 3 и осью вращения (r 3 ) = 20 см = 0,2 метра

F 1 = 10 Ньютон

F 2 = 10 Ньютон

F 3 = 10 Ньютон

Разыскивается: Результирующий момент силы относительно точки C.

Решение:

Момент силы 1:

Στ 1 = (F 1 ) (r 1 sin 90 o ) = (10 Н) (0,3 м) (1) = -3 Нм

Знак минус означает, что момент силы вращает шток по часовой стрелке.

Момент силы 2:

Στ 2 = (F 2 ) (r 2 sin 90 o ) = (10 Н) (0,1 м) (1) = 1 Н · м

Знак плюс означает, что момент силы вращает шток против часовой стрелки.

Момент силы 3:

Στ 3 = (F 3 ) (r 3 sin 30 o ) = (10 Н) (0,2 м) (0,5) = -1 Н · м

Знак минус означает, что момент силы вращает шток по часовой стрелке.

Равнодействующая момента силы:

Στ = Στ 1 + Στ 2 + Στ 3

Στ = -3 + 1 — 1

Στ = -3 Нм

Знак минус указывает, что равнодействующий момент силы вращает шток по часовой стрелке.

5. Три силы F 1 , F 2 и F 3 действуют на стержень, как показано на рисунке ниже. Длина стержня 4 метра. Каков момент силы относительно точки C.

(sin 53 o = 0,8, cos 53 o = 0,6, AB = BC = CD = DE = 1 метр)

Известный:

Ось вращения в точке C.

Сила 1 (F 1 ) = 5 Ньютон

Расстояние между линией действия F 1 с осью вращения (r 1 ) = 2 метра

Force 2 (F 2 ) = 0.4 Ньютона

Расстояние между линией действия F 2 с осью вращения (r 2 ) = 1 метр

Сила 3 ​​(F 3 ) = 4,8 Ньютона

Расстояние между линией действия F 3 с осью вращения (r 3 ) = 2 9607 метров

Разыскивается: Момент силы относительно точки C.

Решение:

Момент силы 1:

τ 1 = F 1 r sin 53 o = (5 Н) (2 м) (0,8) = (10) (0,8) N = 8 Н

Знак плюс означает, что момент силы вращает шток против часовой стрелки.

Момент силы 2:

τ 2 = F 2 r sin 90 o = (0,4 Н) (1 м) (1) = -0,4 Н

Знак минус означает, что момент силы вращает шток по часовой стрелке.

Момент силы 3:

τ 3 = F 3 r sin 90 o = (4,8 Н) (2 м) (1) = -9,6 Н

Знак минус означает, что момент силы вращает шток по часовой стрелке.

Равнодействующая момента силы:

Στ = τ 1 — τ 2 — τ 3 = 8 — 0,4 — 9,6 = 8-10 = 2 Н · м

Знак плюс означает, что момент силы вращает шток против часовой стрелки.

6.Чему равен момент силы вокруг оси вращения в точке O силами, действующими на стержень, как показано на рисунке ниже?

Известный:

Ось вращения в точке О.

Сила 1 (F 1 ) = 6 Ньютон

Расстояние между линией действия F 1 с осью вращения (r 1 ) = 1 метр

Сила 2 (F 2 ) = 6 Ньютон

Расстояние между линией действия F 2 с осью вращения (r 2 ) = 2 метра

Сила 3 ​​(F 3 ) = 4 Ньютона

Расстояние между линией действия F 3 с осью вращения (r 3 ) = 2 метра

Разыскивается: Равнодействующая момента силы относительно точки C

Решение:

Момент силы 1:

τ 1 = F 1 l 1 = (6 Н) (1 м) = 6 Нм

Знак плюс означает, что момент силы вращает шток против часовой стрелки.

Момент силы 2:

τ 2 = F 2 r 2 sin 30 o = (6 Н) (2 м) (0,5) = 6 Нм

Знак плюс означает, что момент силы вращает шток против часовой стрелки.

Момент силы 3:

τ 3 = F 3 l 3 = (4 Н) (2 м) = -8 Нм

Знак минус означает, что момент силы вращает шток по часовой стрелке.

Равнодействующая момента силы:

Στ = τ 1 + τ 2 — τ 3 = 6 + 6-8 = 4 Н.м

Знак плюс означает, что момент силы вращает шток против часовой стрелки.

, пожалуйста, ответьте на этот вопрос 6. Вычислите момент силы P относительно точки A по (a) …

  • В каждом из приведенных ниже случаев вычислите момент силы F относительно точки …

    В каждом из приведенных ниже случаев вычислите момент силы F относительно точки B с помощью следующих подходов: a) Используя плечо момента db) Разложите силу на компоненты x и y c) Поместите силу в D и разделите ее на компоненты г) Позиционируя силу в точке C, и разложив ее на составляющие 12 дюймов.60 30 ° F-25 фунтов A 12 дюймов

  • Пожалуйста, ответьте на это как можно скорее !! Момент достижения цели обучения формулировке вектора силы Использование векторного креста …

    Пожалуйста, ответьте на это как можно скорее !! Момент обучения формулировке вектора силы и вектора Цель обучения Использование векторного векторного произведения для вычисления момента, создаваемого силой или силами, относительно указанной точки на элементе Момент силы F относительно оси момента, проходящей через точку O и перпендикулярной оси плоскость, содержащая o и F, может быть выражена с помощью векторного векторного произведения Mo x F.В правильно построенной декартовой системе координат векторное векторное произведение может быть …

  • 1. Найдите момент относительно точки O за счет силы P. 2. Найдите момент …

    1. Найдите момент относительно точки O за счет силы P. 2. Найдите момент около точки O из-за комбинированного силы Q и R 3. Найдите момент около точки B из-за объединенные силы Q и R 4. Найдите момент около точки А из-за того, что объединенные силы Q и R 5. Одинаковы ли моменты относительно точек O, B, A из-за объединенные силы Q и R.Почему? 6. Не …

  • 1. На конструкцию трубы на рисунке ниже действует сила в точке …

    1. На конструкцию трубы на рисунке ниже действует сила в точке C, как показано. F 2888 + 384ý — 2402) N a. Определите вектор положения от A до C b. Используя векторное произведение, определите момент силы относительно точки A. c. Используя скалярный анализ, компоненты вектора положения и компоненты вектора силы, пересчитайте момент относительно осей x, y и z 300 мм 240 мм…

  • Силовые пары и моменты

    К рычагу прилагаются две параллельные силы 60 Н, как показано. Определите момент пары, образованной двумя силами: а) разделив каждую силу на горизонтальную и вертикальную составляющие и сложив моменты двух полученных пар, б) используя перпендикулярное расстояние между двумя силами и в) суммируя моменты двух сил относительно точки A. Ответ для a, b и c составляет 12,39 Нм.

  • 2.В точке A приложена сила F 9 кН. (A) Вычислите момент F примерно …

    Сила F 2,9 кН применяется в точке A. (a) Вычислите момент F относительно точки O, выразив его как скаляр (положительный, если против часовой стрелки, отрицательный, если против часовой стрелки), и как векторную величину. (b) Определите координаты точек на осях x и y, относительно которых момент F равен нулю. 1,7 m, a 3, b 7. Предположим, xA 1,4 m, yA y, m A (x y a F b —- x, m

  • Используя вектор положения от A до C, вычислите момент относительно сегмента A B из-за силы F.

    Общий процесс (не связанный с рисунком 1) для вычисления момента силы вокруг указанной оси выглядит следующим образом: величина момента относительно отрезка прямой, соединяющего точки P и Q, из-за силы F, приложенной в точке R (с R не на прямой, проходящей через P и Q) может быть вычислено с использованием скалярного тройного произведения, MPQ = uPQ · r × F, где r — вектор положения из любой точки на прямой, проходящей через P …

  • Найдите момент пары, образованной двумя силами (а), разрешив каждую силу…

    Найдите момент пары, образованной двумя силами (a), разделив каждую силу на горизонтальную и вертикальную составляющие и сложив моменты двух полученных пар, (b) используя перпендикулярное расстояние между двумя силами, (c) и суммированием моментов двух сил на штифте. (Ваши ответы должны быть одинаковыми на все три.) (Г) Почему ваш ответ не равен 100 2? 100 фунтов 4 3 2 дюйма 5 …

  • HW5. а) Найдите момент 1.Сила 80 кН относительно точки Б. Выразите свой ответ …

    HW5. a) Найдите момент силы 1,80 кН относительно точки B. Выразите свой ответ в виде стандартного декартова вектора, b) Теперь найдите составляющую этого момента относительно линии от B до A. 1,80 кН 4 м 6 м 2 м

  • HW5. а) Найдите момент силы 1,80 кН относительно точки Б. Выразите свой ответ …

    HW5. a) Найдите момент силы 1,80 кН относительно точки B. Выразите свой ответ в виде стандартного декартова вектора.б) Теперь найдите составляющую этого момента относительно линии от B до A. 1,80 кН 4 м 2 м 6 м 2 м 6 м 4 м.

  • [PDF] Документ — Скачать бесплатно PDF

    Скачать документ …

    ENGR-1100 Введение в инженерный анализ

    Лекция 9

    МОМЕНТ СИЛЫ (СКАЛЯРНАЯ ФОРМУЛЯЦИЯ), ПЕРЕКРЕСТНОЕ ПРОИЗВОДСТВО, МОМЕНТ СИЛЫ (ВЕКТОРНАЯ ФОРМУЛЯЦИЯ) и ПРИНЦИП МОМЕНТОВ Задачи сегодняшнего дня: Студенты смогут: а) понимать и определить момент, и, б) определить моменты силы в 2-D и 3-D случаях.

    Классные занятия: • Викторина по чтению • Приложения • Момент в 2-х измерениях • Момент в 3-х измерениях • Тест на концепцию • Решение групповых проблем • Тест на внимание

    ПРИЛОЖЕНИЯ

    Балки часто используются для перекрытия зазоров в стенах. Мы должны знать, какое влияние сила на балку окажет на опоры балки. Как вы думаете, что происходит в точках A и B?

    ПРИМЕНЕНИЕ (продолжение)

    Плотники часто используют молоток таким образом, чтобы вытащить твердый гвоздь.Каким действием сила FH на ручке тянет гвоздь? Как можно математически смоделировать действие силы FH в точке O?

    МОМЕНТ СИЛЫ — СКАЛЯРНАЯ ФОРМУЛЯЦИЯ (Раздел 4.1)

    Момент силы относительно точки обеспечивает меру тенденции к вращению (иногда называемый крутящим моментом).

    МОМЕНТ СИЛЫ — СКАЛЯРНАЯ ФОРМУЛИРОВКА (продолжение) В двумерном случае величина момента Mo = F d

    Как показано, d — перпендикулярное расстояние от точки O до линии действия силы. .В 2-D направление MO — либо по часовой стрелке (CW), либо против часовой стрелки (CCW), в зависимости от тенденции к вращению.

    МОМЕНТ СИЛЫ — СКАЛЯРНАЯ ФОРМУЛЯЦИЯ (продолжение) F Например, M = F d и направление a

    b O

    O

    — против часовой стрелки. d

    Часто проще определить MO, используя компоненты F, как показано.

    Fy

    b

    a

    F Fx

    O

    Тогда MO = (FY a) — (FX b).Обратите внимание на разные обозначения условий! Типичное соглашение о знаках для момента в 2-D гласит, что против часовой стрелки считается положительным. Мы можем определить направление вращения, представив тело, закрепленное в точке O, и решив, в каком направлении оно будет вращаться из-за силы.

    ПРОДУКТ ВЕКТОРНОГО КРЕСТА (Раздел 4.2) Хотя определение момента силы в 2-D несложно, если известно перпендикулярное расстояние d, определение перпендикулярных расстояний может быть затруднено, особенно когда вы работаете с силами в трех измерениях.Итак, существует более общий подход к нахождению момента силы. Этот более общий подход обычно используется при работе с трехмерными силами, но может быть использован и в двумерном случае. Этот более общий метод определения момента силы использует векторную операцию, называемую перекрестным произведением двух векторов.

    ПЕРЕКРЕСТНОЕ ПРОИЗВОДСТВО (Раздел 4.2)

    В общем случае, перекрестное произведение двух векторов A и B дает другой вектор, C, т. Е. C = A  B. Величина и направление результирующего вектора могут быть записаны как C = A  B = AB sin  uC Как показано, uC — это единичный вектор, перпендикулярный векторам A и B (или плоскости, содержащей векторы A и B).

    ПЕРЕКРЕСТНОЕ ПРОИЗВЕДЕНИЕ (продолжение) Правило правой руки — полезный инструмент для определения направления вектора, полученного в результате перекрестного произведения. Например: i  j = k Обратите внимание, что вектор, пересекающийся сам с собой, равен нулю, например, i  i = 0

    ПЕРЕКРЕСТНОЕ ПРОИЗВЕДЕНИЕ (продолжение) Кроме того, перекрестное произведение может быть записано как определитель.

    Каждый компонент можно определить с помощью определителей 2  2.

    МОМЕНТ СИЛЫ — ФОРМУЛИРОВКА ВЕКТОРА (Раздел 4.3)

    Моменты в 3-D могут быть вычислены с использованием скалярного (2-D) подхода, но это может быть сложно и требует много времени.Таким образом, часто проще использовать математический подход, называемый векторным кросс-произведением.

    Используя векторное произведение, MO = r  F. Здесь r — вектор положения от точки O до любой точки на линии действия F.

    МОМЕНТ СИЛЫ — ФОРМУЛЯЦИЯ ВЕКТОРА (продолжение) Итак, используя перекрестное произведение, момент может быть выражен как

    . Расширяя приведенное выше уравнение с использованием 2  2 определителей (см. раздел 4.2), мы получаем (единицы измерения — Н — м или фунт — фут) MO = (ry FZ — rZ Fy) i  (rx Fz — rz Fx) j + (rx Fy — ry Fx) k Физический смысл приведенного выше уравнения становится очевидным, если рассматривать компоненты силы отдельно и использовать двумерную формулировку.

    ПРИМЕР I Дано: к раме приложено усилие 100 Н. Найти: момент силы в точке O. План:

    1) Определите силу 100 Н по осям x и y. 2) Определите MO, используя скалярный анализ для двух компонентов силы, а затем сложите эти два момента вместе.

    ПРИМЕР I (продолжение)

    Решение +  Fy = — 100 (3/5) N +  Fx = 100 ( 4/5) Н + МО = {- 100 (3/5) Н (5 м) — (100) (4/5) Н (2 м)} Н · м = — 460 Н · м

    или 460 Н · М CW

    ПРИМЕР II Дано: F1 = {100 i — 120 j + 75 k} фунт F2 = {- 200 i +250 j + 100 k} фунт o

    Найти: Результирующий момент сил относительно точки O.План: 1) Найдите F = F1 + F2 и rOA. 2) Определите MO = rOA  F.

    ПРИМЕР II (продолжение) Решение: сначала найдите вектор результирующей силы FF = F1 + F2 = {(100-200) i + (-120 + 250) j + (75 + 100) k} фунт = {-100 i +130 j + 175 k} lb Найдите вектор положения rOA rOA = {4 i + 5 j + 3 k} ft. Затем найдите момент, используя векторное векторное произведение. i 4

    jk MO = 5 3 = [{5 (175) — 3 (130)} i — {4 (175) — 3 (-100)} j + {4 (130) — 5 (-100)} k] фут · фунт -100 130 175

    = {485 i — 1000 j + 1020 k} фут · фунт

    ВИКТОРИНА ПО ЧТЕНИЮ F = 12 N

    1.Каков момент силы 12 Н относительно точки A (MA)? A) 3 Н · м D) (12/3) Н · м

    B) 36 Н · м

    C) 12 Н · м

    E) 7 Н · м

    2. Момент силы F о точка O определяется как MO = ___________. A) rx F

    B) F xr

    C) r • F

    D) r * F

    • A

    d = 3 м

    КОНЦЕПЦИЯ ВИКТОРИНА 1. Если сила величиной F может быть приложена в четырех различные двухмерные конфигурации (P, Q, R и S), выберите случаи, в результате которых на гайке будут установлены максимальные и минимальные значения крутящего момента.(Макс, Мин). A) (Q, P)

    B) (R, S)

    C) (P, R)

    D) (Q, S)

    SRP

    Q

    РЕШЕНИЕ ПРОБЛЕМЫ ГРУППЫ I yx

    Дано : К молотку прилагается сила в 20 фунтов. Найдите: момент силы в точке A. План: Поскольку это двумерная задача: 1) Устраните силу в 20 фунтов по осям x и y ручки. 2) Определите MA с помощью скалярного анализа.

    ГРУППА РЕШЕНИЕ ПРОБЛЕМ I (продолжение) yx

    Решение: +  Fy = 20 sin 30 ° lb +  Fx = 20 cos 30 ° lb

    + MA = {- (20 cos 30 °) lb (18 дюймов) — (20 sin 30 °) фунт (5 дюймов)} = — 361.77 фунт · дюйм = 362 фунт · дюйм (по часовой стрелке или по часовой стрелке)

    ГРУППА РЕШЕНИЕ ПРОБЛЕМ II Дано: Показанные сила и геометрия. Находка: Момент F около точки A План: 1) Найдите F и rAC. 2) Определите MA = rAC  F

    РЕШЕНИЕ ПРОБЛЕМЫ ГРУППЫ II (продолжение) Решение: F = {(80 cos30) sin 40 i + (80 cos30) cos 40 j  80 sin30 k} N = {44,53 i + 53,07 j  40 k} N rAC = {0,55 i + 0,4 j 0,2 k} m Найдите момент, используя векторное произведение. MA =

    i

    j

    k

    0,55 0,4  0.2 44,53 53,07  40

    = {-5,39 i + 13,1 j +11,4 k} Н · м

    ВОПРОС ВНИМАНИЯ 10 N

    3 м

    P

    2 м

    5N

    1. Использование направления против часовой стрелки в качестве положительного , чистый момент двух сил относительно точки P равен A) 10 Н · м B) 20 Н · м C) — 20 Н · м D) 40 Н · м E) — 40 Н · м 2. Если r = { 5 j} m и F = {10 k} N, момент

    rx F равен {_______} Н · м. A) 50 i

    B) 50 j

    D) — 50 j

    E) 0

    C) –50 i

    (решено) — чтобы иметь возможность рассчитать момент силы относительно заданного… — (1 ответ)

    Чтобы иметь возможность вычислить момент силы относительно указанной оси. Когда момент силы вычисляется относительно точки, момент и его ось всегда перпендикулярны плоскости, которая содержит силу и момент amp. В некоторых случаях важно найти составляющую этого момента вдоль указанной оси, которая проходит через точку. Помимо скалярного анализа. Ma, величина момента силы вокруг указанной оси, может быть найдена с помощью тройного скалярного произведения M_a = middot (r умноженное на F) = | u_a1 r_x F_x u_ay r_y F_y u_a1 r_z F_z | Эту проблему можно решить, используя либо скалярный анализ, либо тройное скалярное произведение, и вы можете использовать другой метод, чтобы проверить свой ответ.Когда указанная ось — это x. у. или ось z, тройное скалярное произведение значительно упрощается. Тройное скалярное произведение можно использовать для нахождения составляющей момента силы вокруг любой оси, если найден единичный вектор u вдоль указанной оси. Используя длинный гаечный ключ из Части A., болт теперь откручен на длину 1,00 дюйма. Сила F = {-33,7i + 9,00j + 5,00k} фунтов все еще действует на гаечный ключ в точке B. Что такое M_x . момент силы в точке O относительно оси x? Выразите свой ответ численно в фунт-футах с точностью до трех значащих цифр.Старый деревянный телефонный столб с единственной поперечиной был сорван во время недавнего шторма. Чтобы временно стабилизировать телефонный столб, ремонтная бригада прикрепила кабель от конца поперечины к кольцевой стойке, закрепленной в земле. Если начало 0 системы координат расположено там, где полюс встречается с землей, точка, в которой поперечина прикрепляется к столбу, равна A (- 2,10, 3,00, 24,5) футов, а точка, где кабель прикрепляется к концу. поперечины B (-2.10, -2,00, 27,5) футов Направление троса составляет {1,00i — 1,00j — 1,00k}, а его натяжение составляет 155 фунтов. Что такое M_a. момент, который имеет тенденцию перекручивать телефонный столб из-за натяжения кабеля, приложенного в точке B? Выразите свой ответ численно в фунт-футах с точностью до трех значащих цифр. .

    alexxlab

    Добавить комментарий

    Ваш адрес email не будет опубликован. Обязательные поля помечены *